You are on page 1of 78

Reading comprehension practice

This file contains 30 reading comprehension passages with


answered and explained questions.

Good luck on your test.































On May 5th, 1997, the European edition of Business Tech Magazine led with
Hoffmans cover story "Internet Communities: How They're Shaping Electronic
Commerce". This cover story highlights the extent to which the term virtual
community has become almost synonymous with various forms of group-CMCs
(computer mediated communication), including email-list forums, chat-systems
such as IRC, web-based discussion areas and usenet news-groups. There was no
debate in the Business Tech Magazine article as to whether the group-CMC
discussions are really 'communities', rather how community as opposed to
content can be used to encourage people to return to a particular part of
cyberspace for commercial gain. In a similar vein, Simpson and Armstrong in
"Internet Gain" argue that ignoring virtual communities would be a great loss of a
marketing tool for businesses. They define virtual communities as computer
mediated space where there is an integration of content and communication with
an emphasis on member-generated content.
Not all virtual community commentators agree with the Spartan position taken by
Hoffman. Rheingold, one of the prime popularizers of the term virtual community,
provides us with a more emotive definition in his book The Virtual Community:
Homesteading on the Electronic Frontier. According to Rheingold, "virtual
communities are social aggregations that emerge from the Net when enough
people carry on those public discussions long enough, with sufficient human
feeling, to form webs of personal relationships in cyberspace". Rheingold's
definition is extremely popular and has been quoted in many discussions about
virtual communities. As discussed below, for social scientists, particularly
sociologists, Rheingold's definition raises many issues, especially concerning the
notion of community. This is because Rheingold argues via a variety of analogies
from the real world such as homesteading that virtual communities are indeed
new forms of "community". In fact, Rheingold implies that virtual communities
are actually "a kind of ultimate flowering of community". Moreover, Rheingold
maintains that whenever computer mediated communications technology
becomes available, people inevitably create communities with it. Rheingold can
thus be labeled as a technological determinist as he holds that there is a
predictable relationship between technology and people's behavior.
The debate over the validity of Rheingold's position has raised doubts about the
existence of virtual communities and the appropriate use of the term. Weinreich
claims that the idea of virtual communities must be wrong because community is
a collective of kinship networks that share a common geographic region, a
common history, and a shared value system, usually rooted in a common religion.
In other words, Weinreich rejects the existence of virtual communities because
group-CMC discussions cannot possibly meet his definition. In Weinreich's view,
anyone with even a basic knowledge of sociology understands that information
exchange in no way constitutes a community.

1. The primary purpose of the passage is to
(A) suggest an alternate definition for the term virtual communities
(B) challenge the validity of group-CMCs in the virtual community
(C) discuss whether group-CMCs constitute real communities
(D) present two opposing hypotheses and presents research and evidence to
support them
(E) emphasize the unsuitability of traditional definitions of community in light of
the recent establishment of virtual communities on the Net

The best answer is C. The passage begins by presenting the viewpoint of those
that believe that group-CMCs are true communities and continues by presenting
an opposing view. D is incorrect because definitions, not hypotheses are
discussed in the passage.

2. According to Simpson and Armstrong, virtual communities
(A) are not as effective as content in encouraging people to return to a particular
part of the Internet.
(B) emphasize attracting new members through their use of absorbing content
(C) are not really communities, but simply group-CMC discussions
(D) has become almost synonymous with various forms of group-CMCs
(E) are an invaluable marketing tool for businesses.
The best answer is E. According to Simpson and Armstrong ignoring virtual
communities would be a loss. In other word, virtual communities are invaluable.
The word invaluable means very useful.

3. Weinreich rejects Rheingolds inclusion of virtual communities in the definition
of communities for all of the following reasons EXCEPT
(A) virtual communities do not usually exchange information
(B) virtual communities do not usually share the same territory
(C) virtual communities do not usually share values
(D) virtual communities do not usually share a common history
(E) virtual communities are not usually rooted in a common religion
The best answer is A. The question asks you to identify what does NOT stop
Weinreich from considering a virtual community a real community. An exchange
of information is not problematic for Weinreich.
4. The author says `there was no debate in the Business Tech Magazine article to
emphasize which of the following points?
(A) It is not content but community that can be used to entice the public to go
back to a commercial website
(B) It is not community but content that can be used to entice the public to go
back to a commercial website
(C) It would be a great loss of a business marketing tool if virtual communities
were ignored.
(D) There are various forms of group-CMCs, including email-list forums, chat-
systems, web-based discussion areas and usenet news-groups.
(E) Business Tech Magazine had already made the assumption that group-CMCs
are communities.
The best answer is E. The passage discusses whether group-CMCs are
communities. Business Tech Magazine does not debate this issue because it has
already made the assumption that they are.





















Take a very commonplace, often discussed and critical topic: Are we detecting a
greenhouse effect, and related to this, is it exacerbated by "homogenic factors," i.e.,
human actions? Most would be inclined to give a positive answer to both of these
questions. But, if pushed, what would be the evidence, and how well grounded would
it be for such affirmations?

Within scientific communities and associated scientifically informed circles, the
answers have to be somewhat more ambiguous, particularly when rigorous questions
concerning evidence are raised. Were scientific truth to be a matter of consensus, and
some argue that scientific truth often turns out to be just that, then it is clear that there
is beginning to be a kind of majority consensus among many earth science
practitioners that the temperature of the Earth, particularly of the oceans, is indeed
rising and that this is a crucial indicator for a possible greenhouse effect.

Most of these scientists admit that the mean oceanic temperature has risen globally in
the last several decades. But this generalization depends upon how accurate
measurements may be, not just for samples, but also for the whole Earth. Hot spots,
for example the now four year old hot spot near New Guinea which is part of the El
Nio cycle, does not count by itself because it might be balanced by cold spots
elsewhere. And the fact of the matter is that "whole earth measurements" are still rare
and primitive in the simple sense that we simply do not have enough thermometers
out. Secondly, even if we had enough thermometers, a simply synchronic whole earth
measurement over three decades is but a blip in the diachronic history of ice age
cycles over the last tens of thousands of years. Thirdly, even if we know that the
earth is now heating up, has an ever increasing ozone hole, and from this strange
weather effects can be predicted, how much of this is due to homogenic factors, such
as CFCs, CO2 increases, hydrocarbon burning, and the like? Is it really the case, as
Science magazine claimed in l990, "24% of greenhouse encouraging gases are of
homogenic origin"?

1. In this passage the author is primarily interested in

(A) whether scientific truths are simply a matter of consensus
(B) determining how well established the greenhouse effect is and to what degree it is
worsened by human actions
(C) whether the hot spot El Nio is balanced elsewhere by cold spots
(D) determining if most scientists would be inclined to give a positive answer to the
question of whether there is a greenhouse effect and if it is worsened by human
actions
(E) making a simple synchronic whole earth measurement more than a blip in the
diachronic history of Ice Age cycles over the last tens of thousands of years.

The best answer is B. The author questions the claim that there is indeed a
greenhouse effect that is made worse by human actions. (A) is too general an answer,
while (D) is too specific. (D) is wrong because it is probing whether scientists agree,
not whether there the phenomenon actually exists.








2. The author of the passage would be most likely to agree with which of the
following statements about the greenhouse effect?

(A) 24% of greenhouse encouraging gases are of homogenic origin.
(B) there is a greenhouse effect that is exacerbated by homogenic factors.
(C) The ozone hole is increasing due to homogenic factors, such as CFCs, CO2
increases, hydrocarbon burning, and the like.
(D) One can determine if mean oceanic temperatures have risen globally in the last
several decades only if measurements of ocean temperatures are precise.
(E) Hot spots, such as the El Nio cycle, should not be counted as a factor in the
greenhouse effect.

The best answer is D. Scientists are basing their claims on global warning on rising
ocean temperatures. One can tell if temperatures have in fact risen only by measuring
them correctly.


3. It can be inferred from the passage that

(A) we cannot be certain that strange weather effects are a result of the earth heating
up and an ever increasing ozone hole
(B) the greenhouse effect is the most widely discussed topic in the scientifically
informed circles
(C) If the temperature of the oceans has ceased to rise at an ever increasing rate, then
the rate of global warming has increased
(D) strange weather effects have been shown to be due to the diachronic effects of
hydrocarbon burning and not to increases in CFC.
(E) Strange weather effects are caused by the increase use of CFCs, CO2, and similar
gasses.

The best answer is A. The author is questioning the cause and effect relationship
between the increasingly large ozone hole and global warming, as well as cause and
effect relationship between global warming and strange weather effects.

4. The authors claim that, a simply synchronic whole earth measurement over three
decades is but a blip in the diachronic history of ice age cycles over the last tens of
thousands of years would be strengthened if the author

(A) indicated the minimum number of thermometers necessary for a whole earth
measurement
(B) described the factors that precipitated the start of a new ice age
(C) compare synchronic whole earth measurements with diachronic whole earth
measurements
(D) proved that the mean number of years required to detect significant changes in
weather patterns is greater than thirty
(E) specified the exact location and quantity of thermometers placed by scientists
around the globe

The best answer is D. If one knows that change can be detected only after much more
than thirty years, then measurements taken over a thirty year period is insignificant


































An Australian group named Action Council on Smoking and Health (ACSH) has
recently lobbied to make warnings on cigarette packets more graphic. The council
proposed that striking visual photos of diseased organs should be put on at least
50% of outside packaging, in conjunction with health warnings outlining smoking
hazards enumerated in a separate leaflet placed inside the cigarette packet. The
ACSH claim that bland and ineffectual warnings like "Smoking is a health hazard"
currently found on cigarette packets are not nearly sufficient.


Substituting those inadequate admonitions with explicit photos will provide a
powerful visual stimulus to help smokers relinquish their habit. The current
cautions on cigarette packets have little or no impact on smokers who have
grown immune to the warnings that focus on abstract tobacco related risks and
illnesses from which smokers can easily disassociate themselves. The proposed
new tactics would concentrate on the perspective of the individual smoker
through a demonstration of what is occurring in his body each time he reaches for
a cigarette, rather than a generic cautionary word of advise.


The ACSH cited the results of recent studies conducted by psychologists at
McKean University confirming that evidence related to one's own experience is
more effective at influencing future behavior than a presentation of facts and
figures. An further rationale for the addition of pictures to cigarette packages is
the finding that smokers handle their packets 20-30 times a day, on average,
thus, if graphic pictures on cigarette packets were introduced, smoker would have
20-30 chances to face the harsh reality of what damage they are doing to
themselves each time they light up.


Even more essential than the pictures on the outside label, ACSH strongly
advocate including warnings and helpful information in a leaflet inserted into the
packet of cigarettes. Even an analgesic, ACSH adds, found in every bathroom
cabinet has all possible side effects enumerated in the insert. How much more
imperative is it then when the substance in question is tobacco, a dried weed that
contains highly noxious nicotine that society still accepts even though it kills one
of every two of its users.


Fundamentally, what is at stake here is consumer rights. Smokers should know
what substances they are inhaling, and what damage they are inflicting to their
bodies, though surprisingly, even today, many do not. For this reason alone, the
recommendation for more graphic pictures and warnings on cigarette packets,
which many seem excessive, is being seriously considered.

1. It can be inferred from the passage

(A) That cigarette manufacturers would comply with regulations ordering them to
add graphic pictures of diseased organs to their outside packaging.

(B) That society will not continue to condone smoking if it is proven even more
dangerous than was previously assumed.

(C) That smoking cigarettes causes damage to the internal organs of the body.

(D) That if the written warnings were less bland and ineffectual, smokers would
not take more notice of them.

(E) That smokers look at their cigarette packages each time they take out a
cigarette.
The best answer is C. We do not have information about (A) (B) and (D) from
the passage. (E) is incorrect because the passage claims that smokers have an
opportunity to look at their cigarette packages, not that they actually do.

2. The author cites studies conducted at McKean University to account for why

(A) A presentation of facts and figures is more effective at influencing future
behavior than evidence related to one's own experience.

(B) A presentation of facts and figures is less effective at influencing future
behavior than evidence related to one's own experience.

(C) Evidence related to one's own experience has a more long-lasting effect than
future behavior.

(D) The ACSH claim that graphic visual pictures of diseased organs would not be
more effective than stating facts about the consequences to the body of long-
term smoking.

(E) The ACSH claim that graphic visual pictures of diseased organs would not be
less effective than stating facts about the consequences to the body of long-
term smoking.

The best answer is E. The author mentions the study as evidence presented by
the ACSH to back their claim that visual pictures would be more effective than
the present warning found on cigarette boxes.

3. Which of the following, if true, would be most useful in supporting the claims
made by the ACSH?

(A) There is firm evidence that information communicated in a textual format is
more convincing than the same information conveyed in the form of visual
depictions.

(B) There is firm evidence that information conveyed in the form of visual
depictions is more convincing than the same information communicated in a
textual format.

(C) A study of over 3000 individuals shows a statistically significant relationship
between levels of nicotine in cigarettes and pulmonary damage.

(D) A study of over 3000 individuals shows a statistically significant relationship
between smoking and pulmonary damage.

(E) A survey reveals that 79% of smokers look at their cigarette packages when
taking out a cigarette.

The best answer is B. If smokers were more convinced of the dangers of smoker
by pictures than by text, they would be more likely to be influenced by the
pictures that the ACSH is proposing.

4. The passage does NOT state which of the following about smoking warnings.

(A) Current graphic warnings are effective.

(B) The addition of graphic warnings would be an impetus to smokers to
relinquish their habit.

(C) Current written warnings are not adequate.

(D) Current written warnings are not effective.

(E) Current written warnings are not as exhaustive as those that accompany
common analgesics.

The best answer is A. The passage does not say that current graphic warnings
are effective since there are currently no graphic warnings, only written ones.





















Gene therapy offers a new treatment paradigm for curing human disease. Rather than
altering the disease phenotype by using agents that interact with gene products, or are
themselves gene products, gene therapy can theoretically modify specific genes
resulting in disease cure following a single administration. Initially gene therapy was
envisioned for the treatment of genetic disorders, but is currently being studied for use
with a wide range of diseases, including cancer, peripheral vascular disease, arthritis,
Neurodegenerative disorders and other acquired diseases.

Certain key elements are required for a successful gene therapy strategy. The most
elementary of these is that the relevant gene be identified and cloned. Upon
completion of the Human Genome Project, gene availability will be unlimited. Once
identified and cloned, the next consideration must be expression of the gene.
Questions pertaining to the efficiency of gene transfer and gene expression remain at
the forefront of gene therapy research, with current debates revolving around the
transfer of desired genes to appropriate cells, and then to obtaining sufficient levels of
expression for disease treatment. With luck, future research on gene transfer and
tissue-specific gene expression will resolve these issues for the majority of gene
therapy protocols.

Other important considerations for a gene therapy strategy include a sufficient
understanding of the pathogenesis of the targeted disorder, potential side effects of the
gene therapy treatment, and a more in depth understanding of the target cells which
are to receive gene therapy.

Gene transfer vector is the mechanism by which the gene is transferred into a cell.
Currently there are at least 150 clinical gene therapy protocols worldwide. Since the
approval process for these protocols is not as public outside the U.S., it is difficult to
ascertain the exact number of worldwide protocols. As of December 1995, 1024
patients had been treated with either a gene transfer or gene therapy protocol. Much
controversy exists regarding how many of these patients have benefited from their
gene therapy, and no one has yet been cured.

Public controversy in the field of human gene therapy is driven by several factors.
Ordinary citizens as well as scientists easily understand the enormous potential of
gene therapy, but the former may not appreciate all the pitfalls and uncertainly that lie
in the immediate future. The financial interests of biotechnology firms and, some have
asserted, the career interests of some gene therapists have encouraged extravagant, or
at least overly optimistic public statements about contemporary gene therapy. In spite
of the proliferation of protocols, the actual number of patients treated remains small,
and only one genuinely controlled study of human gene therapy has been published as
of this date.


1. In the passage, the author anticipates which of the following as a possible obstacle
to the introduction of gene therapy to mainstream medicine?

(A) Overly optimistic public statements given by scientists who have a vested interest
(B) The general publics difficulty in grasping gene therapys vast potential.
(C) Unchecked financial interests of biotechnology firms.
(D) The relatively small number of controlled studies of human gene therapy
published as of this date.
(E) Hazards of which the general public is currently unaware.
The best answer is E. The passage states that both ordinary citizens and scientists
understand potential of gene therapy, but the former, i.e. the general public may not
appreciate all the pitfalls and uncertainty that lie in the immediate future.

2. All of the following are mentioned in the passage as elements that are required for a
successful gene therapy strategy EXCEPT:

(A) Identifying the relevant gene
(B) Expressing the relevant gene
(C) Determining the side effects of the relevant gene
(D) Understanding of the pathogenesis of the targeted disorder
(E) Gaining and a more in depth understanding of the target cells which are to receive
gene therapy.

The best answer is C. One must determine the side effects of the relevant gene therapy
treatment, and not of the relevant gene itself.


3. The authors attitude toward the gene therapy as a future cure for cancer,
peripheral vascular disease, arthritis, Neurodegenerative disorders and other acquired
diseases is

(A) Indifference
(B) Disapproval
(C) Amusement
(D) Cautious optimism
(E) Censure

The best answer is D. In the opening sentence, the author optimistically states that
gene therapy offers a new treatment paradigm for curing human disease. However, in
the fifth paragraph the author mentions some of the problems.


4. The primary function of the fifth paragraph is to

(A) Explain effects
(B) Recommend actions
(C) Identify problems
(D) Evaluate solutions
(E) Warn of consequences

The best answer is C. The fifth paragraph is devoted to identifying various problems
in the field today, such as the exaggerated claims made by biotechnological firms with
vested financial interests.



Unlike the United States with its generalissimo politicians - Washington, Jackson,
Grant, and Eisenhower- the martial arts have been conspicuously absent from
Canadian politics. The exception to the rule is former Prime Minister Pierre Elliot
Trudeau, who became the first Canadian leader to bring a gunslinger ethos to
Canadian politics. Trudeau introduced Canada to the refined art of single combat; it
was the politics of doing it my way. Single-combat confrontation implied much
more than the renegade in power did, and far less than the tricks of William Lyon
Mackenzie King, prime minister intermittently between 1921 and 1948.

Trudeaus unique background prepared him for the role of authoritarian leader he
would assume later in life. Born on October 18, 1919, Trudeau lived in French-
speaking Montreal, but heard English at home from his mother, making it easy for the
young politician to appeal to all sectors of Canada, a bilingual country. As a young
man, he walked and cycled through Europe, finding himself on occasion on the wrong
side of the bars in foreign jails. By 1940, Trudeau entered the law faculty at the
University of Montreal. As a student he enlisted in the Canadian Officers Training
Corps, where he was given a commission as a lieutenant, a rank he held until his
retirement in 1947. Trudeau, a renowned sportsman, held a brown belt in karate,
knew how to skin dive and could descend 150 feet off a cliff with ease. He continued
performing flamboyant physical feats even in later life as Canadas fifteenth prime
minister, astounding Canadians with his prowess. The publics adoration made it
possible for him to practice his personal brand of do it my way politics, initiating
profound and long-lasting changes to his country.

Other leaders would never have undertaken to deal with such taboo issues as divorce,
abortion and homosexuality matters likely to infuriate conservative Canada from
coast to coast. Even the powerful Mackenzie King dared not touch any of the three,
though Trudeau tackled them together in an omnibus bill as Minister of Justice under
Lester B. Pearson. His reason for loosening legislation on these issues was, as he put
poetically put it, "The state has no business in the bedrooms of the nation." The
myths-makers have it that this was Trudeau's first deliberate gun slinging move,
performed with the ultimate goal of attaining national leadership. Contrary to popular
belief, Trudeau had no leadership aspirations at the time; all he had was a passion for
combat that eclipsed other religious considerations. Trudeau instigated far-reaching
changes in legislation governing divorce, abortion and homosexuality that have had a
major impact on Canada, shaping the country into what it is today.

1. The primary focus of the passage is on which of the following?
(A) Comparing two Canadian prime ministers and contrasting their personal style of
leadership
(B) Describing the leadership style of one of Canadas prime ministers
(C) Evaluating the success of the leadership style of one of Canadas prime ministers
(D) Summarizing the contribution of one of Canadas prime ministers
(E) Tracing the long-term impact of legislation put forward by one of Canadas prime
ministers

The best answer is B. The passage describes the doing it my way type leadership
style of former Prime Minister Pierre Elliot Trudeau.

2. It can be inferred from the passage that former Prime Minister William Lyon
Mackenzie King
(A) was opposed to abortion and put forward legislation making it illegal
(B) was opposed to abortion but did not put forward legislation making it illegal
(C) was in favor of abortion and put forward legislation making it legal
(D) was in favor of abortion but did not put forward legislation making it legal
(E) did not put forward legislation making abortion legal
The best answer is E. The passage states that Mackenzie King dared not touch the
issue of abortion, so it can be inferred from the passage that he did not put forward
any abortion legislation. The passage provides no information on what Mackenzie
Kings opinions were on the subject.

3. The author of the passage mentions Trudeaus accomplishments in sports primarily
in order to
(A) Explain the source of Trudeaus physical stamina
(B) Illustrate that he had earned the adoration of the Canadian public
(C) Contrast it to his personal brand of do it my way politics
(D) Provide one reason why he was able to single-handedly push through legislation
that should normally have shocked conservative Canada
(E) Provide one reason why he was able to single-handedly thwart legislation that
should normally have shocked conservative Canada

The best answer is D. One of the ways Trudeau won the hearts and minds of the
Canadian public was by astounding them with his physical prowess. Because the
public was so awed by Trudeau as an individual, he was able to pass legislation
considered progressive.

4. The quotation "The state has no business in the bedrooms of the nation," is most
probably used to
(A) present the opinion that the state should have less of a say in issues that are
essentially not public matters
(B) present the opinion that the state should have less of a say in issues that are
essentially public matters
(C) present the opinion that the state should have more of a say in issues that are
essentially not personal matters
(D) provide an example of Pierre Elliot Trudeaus flamboyant style that he used to
captivate the Canadian public
(E) contrast Pierre Elliot Trudeaus flamboyant style with that of Lester B. Pearson
The best answer is A. Matters that concern the bedrooms of the nation are private
matters. Trudeau sought to reduce government control in such matters. Answers (B)
and (C) say the opposite, namely that government control in public matters







Art is visible. However, everything one sees is filtered through certain conditions,
some of them historical, and others, natural. The historical conditions include the
material which is used oil, colors, and the canvas; second, a certain style, i.e., a
system of rules by which things visible are submitted a priori. There can be a general
style, for example, the style of Impressionism, or a particular style, for example, the
individual ways in which two painters, both impressionists, paint. The natural
conditions include certain unchanging psychological laws of sight, for instance, the
effects of colors or optical illusions.
The conditions of art are nothing but a particular way of interpreting reality. To
understand this, one can examine the difference between the classical Greek and the
classical Egyptian styles. For the Greeks, the reality of the visible was given by the
perspective and the situation in which the object appears; for that reason they
presented a person in his individual movements. For the Egyptians, however, this was
only the appearance of a transitory moment, which, according to their beliefs, was not
real. Therefore, the Egyptians searched for the permanent essence and the typical
character in their depiction of an object. For the Egyptians, Greek art was an illusion;
for the Greeks, on the other hand, Egyptian art was unrealistic constructivism.
The way in which reality appears in art must not be regarded on its own. It is affected
by many other systems of recognizing reality, including the political, religious,
economic, intellectual, and social in short, all the phenomena of human life.
Moreover, art is always of a certain epoch, with its particular conception of reality.
Thus, when discussing, for example, the art of ancient myth, of medieval Christianity,
or that of the technological age, one must be aware that myth, Christianity, or
technology was the most salient feature of the epoch.
It is paradoxical to understand art as some kind of copy of the fields of experience
connected with it. So, for example, it is meaningless for the work of art as such if one
compares the landscape of a painting with the landscape, which served the artist as his
model. Even if the artist had tried to make what he painted as similar as possible to
the model he used, the landscape which he saw is only the matter from which
something completely different emerges since he has submitted its view to the a priori
conditions of art: namely to the material used (colors, canvas, etc.), to his style, and
even to the fact that he paints on a flat surface. Thus one must contemplate a work of
art by itself. Even if it is connected to other fields of experience it nevertheless
displays something unique which appears in that piece of art and there alone.

1. According to the passage, classical Egyptians did not present a person in his
individual movements

(A) Because the Greeks believed that the reality of the visible was given by the
perspective and the situation in which the object appears
(B) Because the Greeks did not believe that the reality of the visible was given by the
perspective and the situation in which the object appears
(C) Because the Egyptians believed that the reality of the visible was given by the
perspective and the situation in which the object appears
(D) Because the Egyptians did not believe that the reality of the visible was given by
the perspective and the situation in which the object appears
(E) Because for the Egyptians, Greek art was an illusion.

The best answer is D. The Egyptians believed that the reality of the visible was not
given by the perspective and the situation in which the object appears. They thought
that this was the appearance of a transitory moment, which was not real.

2. The author mentions which of the following as one of the conditions through
which art is seen?
(A) Impressionism
(B) Optical illusions
(C) Nature
(D) Perspective
(E) Illusions

The best answer is B. The author states in the first paragraph that art is seen through
natural conditions including optical illusions.

3. By asserting that art is filtered through certain conditions (line???), the author
suggests which of the following?

(A) Even if the artist tried to make what he painted as similar as possible to the model
he used, he would not succeed
(B) Even if the artist tried to make what he painted as similar as possible to the model
he used, it would be impossible to critique it
(C) Even if the artist tried to make what he painted as similar as possible to the model
he used, the landscape would be affected by many other systems of recognizing
reality
(D) The way in which reality appears in art influences other systems of recognizing
reality, including the political, religious, economic, intellectual, and social
(E) The way in which reality appears in art is influenced by other systems of
recognizing reality, including the political, religious, economic, intellectual, and
social

The best answer is E. The author states in paragraph three that the way in which
reality appears in art is affected by many other systems of recognizing reality,
including the political, religious, economic, intellectual, and social in short, all the
phenomena of human life.

4. The author cites the example of psychological laws of sight, for instance, the
effects of colors or optical illusions in order to illustrate

(A) Laws of nature, which are not affected by history
(B) Laws of nature, which are affected by history
(C) Laws of nature, which change history
(D) Laws of history, which are affected by nature
(E) Laws of history, which are not affected by nature

The best answer is A. The author details two kinds of conditions, historical and
natural. The natural conditions are said to be unchanging, therefore not affected by
history.


For a generation of suppressed, restless, working-class youths living in 1960 Jamaica,
ska was a medium through which they could find expression. Since its original
appearance, ska has resurfaced twice, each time presenting itself in a different guise to
a new generation of music aficionados. Overcoming its humble beginnings, it has
become one of the twentieth centurys most enduring and influential styles of music.

Since the early 1940's, Jamaica had adopted and adapted many forms of American
musical styles. The predominantly black inhabitants of Jamaica took a liking to
rhythm and blues music, importing a considerable number of American records that
were showcased at dance halls in the early 1960s. Jamaican musicians took up the
elements of rhythm and blues and combined it with traditional Jamaican mento music.
The result was the first wave of ska.

Musically, ska is a shuffle rhythm similar to mento but with even closer ties to rhythm
and blues, placing the accent on the second and fourth beats, often moving in a 12-bar
blues frame. The after beat, played on the piano or strummed by a rhythm guitar,
came to be characteristic of the form. A horn section, usually consisting of trumpets,
trombones, and saxophones, was a vital element. Classic bands, such as the Wailers
wrote songs written about Trench Town (a ghetto), rude boys (street thugs), romance,
and even religious themes. In 1965, ska began to take a backseat to a newly evolved
type of music, called rock steady, which was more dependent than ska had been on
rhythm provided by the bass guitar and drums.

Ska was later exported by traveling Jamaican artists to Great Britain, where it became
known as "blue beat." By the mid 1970's, early British punk bands were infusing
reggae, a style of music that came from rock steady, into their music. Near the end of
the decade, however, there was a resurgence of the influence of ska because of its
upbeat, danceable rhythm. This faster paced ska came to be known as two tone. One
of the essential messages of two-tone ska was the promotion of racial harmony and of
having fun in the face of subjugation.

The third wave of ska began in America around 1990. Bands influenced by the two-
tone ska scene began to use punk and metal music to a greater extent. The
combination, which is much faster than two tone, sounds very different from the
original Jamaican brand of ska.

In its three different waves, ska has given voice to seemingly voiceless, downtrodden
generations. Each time it resurfaces, a new message is taken up, however, the old
messages are never forgotten.

1. The primary purpose of this passage is to

Contrast the musical rhythm of two-tone music with original Jamaican ska from
which it developed
Illustrate various ways in which rhythm and blues has influenced ska music
Outline the influences on the various forms of ska music from its inception in Jamaica
in early 1960s through its third wave in the 1990s
Describe events leading to the inception, rise and final demise of Jamaican ska music
Trace to evolution of ska music from its inception in Jamaica in early 1960s through
its third wave in the 1990s

The best answer is E. The passage follows the development of ska. (D) is incorrect
because the passage does not deal with the demise, or death, of ska music.

2. According to the passage, Ska music has

Been influenced by rhythm and blues, mento and blue beat
Been influenced by rhythm and blues, and has influenced mento and rock steady
Been influenced by rhythm and blues and rock steady and has influenced punk and
metal musicians
Influenced rhythm and blues, reggae and metal musicians.
Been influenced by mento music and has influenced punk and metal musicians.

The best answer is E. Ska music has been influenced, among other things, by mento
music. In paragraph five, it is mentioned that ska musicians have influenced both
punk and metal musicians.

3. Which of the following statements about ska music is supported by information in
the passage?

Rock steady is more dependent than ska on the rhythm provided by the bass guitar
and drums.
Reggae, which counts ska as one of its primary influences, developed only after it was
exported by traveling Jamaican artists to Great Britain
Skas appeal over the last half century has been limited to voiceless, downtrodden
generations.
Two-tone is a faster paced form of ska that developed in the late 1970s
Mento music places the accent on the second and fourth beats, often moving in a 12-
bar blues-frame.

The best answer is D. In paragraph four, it says that this faster paced ska came to be
known as two tone.

4. The passage suggests that two tone music

I. Resurged near the end of the 1970s
II. Influenced bands in America in the 1990s
III. Promoted of racial harmony and of having fun in the face of oppression.

I only
II only
III only
II and III only
I, and II, and III

The best answer is E. (E) is the best choice because all of the facts presented above
are supported by the passage.



Coca-Cola, which sold 10 billion cases of soft drinks in 1992, now finds itself asking,
where will sales of the next 10 billion cases come from? The answer lies overseas,
where income levels and appetites for Western products are at an all time high.

Often, the company that gets into a foreign market earliest dominates that country's
market. Coke patriarch Robert Woodruff realized this and unleashed a brilliant ploy to
make Coke the early bird in many of the major foreign markets. At the height of
World War II, Woodruff proclaimed, Wherever American boys were fighting, they'd
be able to get a Coke. By the time Pepsi tried to make its first international pitch in
the 1950s, Coke had established its brand name along with a powerful distribution
network.

During the last 40 years, many new markets have emerged. In order to tap into these
opportunities, both coke and Pepsi have attempted to find ways to cut through the red
tape that thwarts their efforts to conduct business in these new regions.

One key maneuver in the soda wars occurred in 1972, when Pepsi signed an
agreement with the Soviet Union that made it the first Western product to be sold to
consumers in Russia. This landmark agreement gave Pepsi the upper hand. At present,
Pepsi has 23 plants in the former Soviet Union and is the leader in the soft-drink
industry in Russia. It outsells Coca-Cola by a ratio of 6 to 1 and is seen there as a
local brand, similar to Cokes homegrown reputation in Japan. However, Pepsi has
also encountered some obstacles. An expected increase in brand loyalty for Pepsi
subsequent to its advertising blitz in Russia has not materialized, even though Pepsi
produced commercials tailored to the Russian market and sponsored televised
concerts.

Some analysts believe that Pepsis domination of the Russian market has more to do
with pricing. While Pepsi sells for 250 Rubles (about 25 cents) a bottle, Coca-Cola
sells for 450 Rubles. Likewise, Pepsi sells their 2 liter economy bottle for 1,300
Rubles, while Coca-Colas 1.5 liters is marketed at 1,800 rubles. On the other hand,
Coca-Cola only made its first inroads into Russia 2 years ago. What's more, although
Coca-Cola's bottle and label give it a high-class image, Russians do not perceive
Coca-Cola as a premium brand in the Russian market. Consequently, it has so far
been unable to capture a market share.

1. According to the passage, all of the following have been used to attract customers
to buy a one of the two brands of soft drink mentioned in the passage EXCEPT

Offering soft drinks for a limited time at specially reduced prices
Sponsoring televised concerts
Designing a bottle and label to create a high-class image
Staging an advertising blitz including commercials tailored to the local market
Being the first country to enter a foreign market

The best answer is A. Answers (b), (C), (D) and (E) are all mentioned in the passage
as ploys used by either Pepsi or Coca Cola to attract new customers

2. The passage suggests which of the following about the Russian soft drink market?

Price is an unimportant factor in the Russian soft drink market
Two liter economy bottles are more marketable than 1.5-liter economy bottles,
especially those sporting a high-class image.
One and a half liter economy bottles are more marketable than two liter economy
bottles, if sold at a lower price.
Russian consumers are more likely to purchase a product if the perceive it to be a
local brand
The Russian soft drink market is saturated with local brands.

The best answer is D. The passage states that one of the factors contributing to
Pepsis success in Russia is its perception by the public as a local brand.

3. The primary purpose of the passage is to

Review the marketing history of two soft drink giants
Contrast two different approaches to marketing soft drinks in the global market
Refute the traditional explanation for Pepsis success in the Russian soft drink market
Compare how well two soft drink companies have succeeded in a new foreign market
Explain why two soft drink companies have succeeded in a new foreign market

The best answer is D. The passage mainly compares Pepsis success in a new foreign
market, Russia, with Coca Colas relative failure. (A) is too general. (B) is incorrect
because both companies have the same general approach.

4. Which of the following best describes the relation of the first paragraph to the
passage as a whole?

It poses a question to be answered
It outlines an objective whose attainment will be discussed
It outlines a process to be analyzed
It advances and argument to be disputed
It introduces conflicting arguments to be reconciled

The best answer is B. The first paragraph asks and answers the question of where the
sale of the next 10 billion cases will come from, namely foreign markets. The rest of
the passage discusses ways in which the objective of conquering foreign markets is
accomplished.










With the proliferation of electronic technologies in the latter part of the twentieth
century, many aspects of cultural practice have been redefined. The eradication of
physical boundaries that limit discourse and information access has had profound
effects upon the manner in which we conduct democracy. Yet, opinions strongly
differ over whether or not the growth of electronic networks will result in expanded
democracy. On one side of the debate are anti-utopians who fear that with the
intrusion of the Internet into many facets of life, personal freedom will be impeded
and the existing rift between the "haves" and "have-nots" in society will grow. On the
other side, many 'cyber-utopians' believe that new technologies can eliminate the
democracy of elected representatives with which so many people are dissatisfied. The
Internet, they say, will allow for a true participatory democracy in which citizens can
govern themselves without the interference of bureaucrats and legislators.
Neither of these theories by themselves can fully address the role of democracy in the
age of information. As debates about censorship and encryption have shown,
government regulation of the Internet can result in violations of the basic rights of
speech set forth in the constitution of the United States. Yet, groups that preach Big
Brother theories of paranoia tend to neglect the fact that new technologies can help
balance the injustices of traditional power found in a centralized government. At the
same time, the likelihood of doing away with the present system of democracy in
favor of complete and pure self-governance seems impossible, and likely undesirable.
Both arguments about the future of the way in which discourse will occur highlight
the inherent relationship between communication and democracy. Perhaps a more
useful model for the study of this dynamic can be found in the model of the public
sphere proposed by Jorgen Habermas. In this realm, free and diverse equals come
together to deliberate and discuss pertinent issues without the impediment of external
coercion. The ensuing dialogue transpires in a profoundly democratic forum. The
dispensing of traditional hierarchies that occurs on the Internet appears to make
possible the type of categories necessary for Habermas ideal speech situation to
occur.
However, postmodern critics indicate that the autonomous individual no longer exists
in a world where our identities are constructed as much for us as by us. And indeed,
much of the postmodern notion of self seems to fit closely with reconfigurations of
the subject brought on by electronic technologies. The question that arises then is,
how might the reconfiguration of communication enabled by the Internet work to
create a new form of 'cyber-democracy that better represents citizens' interests?

1. According to the passage, the 'cyber-utopians' mentioned in the passage would
most likely be in favor of which of the following innovations?

Every new legislation would be voted by every registered voter on the Internet
Government would increase the regulation of the Internet to include a curtailing of
politically biased messages
Government would decrease the regulation of the Internet including regulation of
politically biased messages
Discourse in legislative assemblies would be broadcast over the Internet
New technologies would gradually replace all forms of democracy

The best answer is A. According to the passage, 'cyber-utopians' believe that through
using the Internet, there could be a true participatory democracy, meaning that all
citizens, not just elected representatives could make legislative decisions.

2. The passage supports which of the following statements about government
regulation of the Internet?

Government regulation of the Internet can result in infringements upon citizens
constitutional rights of free speech
Government regulation of the Internet can ensure against infringements upon citizens
constitutional rights of free speech
Government regulation of the Internet will make pure self-governance possible
Government regulation of the Internet will promote new technologies that can help
balance the injustices of traditional power
Government regulation of the Internet will eradicate physical boundaries that limit
discourse and information

The best answer is A. As stated in paragraph three, government regulation of the
Internet can result in violations of the basic rights of speech set forth in the
constitution.


3. The author is primarily concerned with

Advocating the use of the electronic technologies to improve democracy
Challenging the assumptions on which a theory of modern democracy is based
Describing events leading to the discovery of democratic uses of electronic
technologies
Explaining the importance of electronic technologies to modern politics
Examining the relationship between Internet communication and democracy

The best answer is E. The answer is not (A) because the author does not reach any
conclusions. (D) is incorrect because it does not discuss modern politics in general.

4. According to the passage, which of the following is considered by postmodern
critics to be a threat to the notion of self?

The interference of bureaucrats and legislators.
The proliferation of electronic technologies.
Reconfigurations of the subject brought on by electronic technologies.
Traditional hierarchies that occur on the Internet.
The impediment of external coercion.

The best answer is C. In the last paragraph, it says that much of the postmodern notion
of self seems to fit closely with reconfigurations of the subject brought on by
electronic technologies.






Men are primarily and secondarily socialized into believing certain characteristics are
definitive in determining their masculinity. These characteristics range from playing
violently to not crying when they are injured. The socialization of masculinity in our
society begins as early as the first stages of infancy, with awareness of adult gender
role differences being internalized by children as young as two years old.
Studies show that advertising imagery equates masculinity with violence by
portraying the trait of aggression as instrumental to establishing their masculinity.
Lee Bowker, who researched the influence of advertisements on youth, asserts that
toy advertisements featuring only boys depict aggressive behavior and that the
aggressive behavior produces positive consequences more often than negative.
Bowker also looked at commercials with boys that contain references to domination.
His results indicated that 68.6% of the commercials positioned toward boys contain
incidents of verbal and physical aggression. However there were no cross gender
displays of aggressive behavior. Interestingly, not one single-sex commercial
featuring girls showed any act of aggression. Bowkers research helps explain that it
is not just the reinforcement of a childs close caretakers that lends legitimacy to
aggressive masculine tendencies but society as a whole, using the medium of
television.
William Pollack, a Harvard clinical psychologist, talks about how males have been
put in a "gender straightjacket" that leads to anger, despair and often violence. Pollack
states that society asks men to put a whole range of feelings and emotions behind a
mask and shames them if they display any emotion. Pollack contends that boys are
shame phobics, even killing, in extreme cases, to avoid dishonor. It appears that the
standard defined by society allows men to express their emotion only through anger.
Ironically, though these rigid stereotypes of what it means to be a man have been
inculcated from an early age, men are often criticized for being one-dimensional in
their behavior and emotions.
Women often verbalize a desire for males to be sensitive and express their emotions.
But male insensitivity is the culmination of a societal indoctrination begun at birth.
Realistically, men are in a damned if they do, damned if they dont situation. If they
fail to show their emotions, they are berated for being detached from the essence of
what constitutes a human being. On the other hand, if a male decides to expose his
emotions, he is often branded effeminate and regarded as inferior to other males who
stick closer to their genders traditional doctrine.

1. According to the passage, the television commercials examined by Bowker

Showed boys in more acts of verbal and physical aggression than of domination
Showed boys in more acts of domination than of verbal and physical aggression
Showed boys in acts of verbal and physical aggression only towards other boys
Showed boys in acts of verbal and physical aggression only towards other girls
Showed boys in acts of verbal and physical aggression towards other boys and girls

The best answer is C. Bowkers research did not find any cross gender displays of
aggressive behavior, i.e. aggression of one gender to another

2. According to Pollack, one of the reasons for male violence is that

Society shames men who display feelings and emotions other than anger
Men kill in extreme cases to avoid dishonor
Men are often criticized for being one-dimensional in their behavior and emotions
Society uses television as a symbol of its desires
Reinforcement from childs close caretakers lends legitimacy to aggressive masculine
behavior

The best answer is A. (B) is incorrect because it does not give a reason for violence.
(C) is a result of the conditioning that leads to violence, not a reason. (D) and (E) are
incorrect because they are not opinions expressed by Pollack.

3. The passage suggests that, when compared with television advertisement featuring
boys, advertisements that had only girls were found

To have more references to domination
To be 68.6% less aggressive
To be remarkably similar in focus and content
To be replete with extensive examples of cross gender aggression
To be void of any acts of aggression

The best answer is E. Bowker found that not one single-sex commercial featuring
girls showed any act of aggression.

4. Pollack uses the term gender straightjacket to emphasize

The narrow range of emotion that society allows men to express
The broad range of emotion that society allows men to express
The danger of anger, despair and violence towards men
The danger of anger, despair and violence perpetrated by men
The wide range of feelings that men actually experience

The best answer is A. Society does not allow men to act in ways it has deemed
inappropriate, hence Pollack considers the male gender to be in a straightjacket.















Juror anonymity was unknown to American common law and jurisprudence in the
countrys first two centuries. Anonymity was first employed in federal prosecutions of
organized crime in New York in the 1980's. Although anonymous juries are unusual
since they are typically only empanelled in organized-crime cases, its use has spread
more recently to widely publicized cases, such as the federal prosecution of police
officers accused of beating Rodney King and the trial of those accused of the 1993
World Trade Center bombing.

In these cases, attorneys selected a jury from a panel of prospective jurors whose
names, addresses, ethnic backgrounds and religious affiliations remained unknown to
either side. This unorthodox procedure, designed to protect jurors from outside
influence and the fear of retaliation, has occasionally been employed in New York
federal courts since the trial of drug kingpin Leroy "Nicky" Barnes. Despite apparent
benefits, critics assail anonymous juries on the grounds that they are an infringement
of the sixth amendment guarantee of an impartial jury and because they present a
serious and unnecessary erosion of the presumption of innocence.

Since many attorneys believe trials are frequently won or lost during jury selection,
any procedure diminishing the role of counsel in the procedure necessitates close
scrutiny and criticism. Opponents of anonymous juries argue that the procedure
restricts meaningful voir dire, (questioning of the jury panel), and thereby undermines
the defendant's sixth amendment right to an impartial jury. Critics also claim that
jurors interpret their anonymity as proof of the defendant's criminal proclivity,
thereby subverting the presumption of innocence.

However, consistent with due process and the sixth amendment, the trial judge may
refuse to ask prospective jurors any questions not reasonably calculated to expose
biases or prejudices relevant to the case. Although addresses and group affiliations
may indicate significant potential for bias, attorneys do not have an unfettered right to
this information in every circumstance. Denying access to these facts may indeed
constrain an attorney's ability to assemble an ideal jury, but it violates no
constitutional right.


1. The primary purpose of the passage is to

Enumerate reasons why anonymous juries are unconstitutional
Discuss whether anonymous juries are an infringement of the sixth amendment
Identify a shortcoming in a scholarly approach to jurisprudence
Define the concept of anonymous juries and explore efforts taken over the last twenty
years to increase their use
Review strategies for ensuring that anonymous juries will not infringe on the
constitutional right to a fair trial of ones peers

The best answer is B. The passage introduces the concept of anonymous juries and
goes on to discuss their constitutionality.

2. It can be inferred from the passage that a jurors ethnic background and religious
affiliation

Is considered by defendants not to have a significant effect on the outcome of their
trials
Is considered by defendants to have a significant effect on the outcome of their trials
Would be unlikely to have a significant effect on the verdict of a trial
Is considered by attorneys likely to have a significant effect on the verdict of a trial
Is considered by attorneys unlikely to have a significant effect on the verdict of a trial
in a widely publicized case

The best answer is D. In paragraph three it states that many attorneys believe trials are
frequently won or lost during jury selection. The passage gives no information on
what defendant think about anonymous juries.

3. One function of the fourth paragraph of the passage is to

Qualify the extent to which a previously introduced viewpoint may be relevant
Expose the flaw in a criticism put forth in a previous paragraph
Introduce information that supports a theory put forth in a previous paragraph
Support an argument in favor of a given interpretation of a situation
Show the chain of reasoning that led to the conclusions of a specific study

The best answer is B. Critics of anonymous juries base their arguments on the fact
that these juries are unconstitutional. In the fourth paragraph, the author explains that
while anonymous juries may not be ideal, they are not unconstitutional.


4. Which of the following, if true, would ensure that anonymous juries are not an
erosion of the presumption of innocence?

Anonymous juries are used in all court cases, regardless of identity of the defendant.
Anonymous juries are used in all court cases involving previously convicted
defendants.
Anonymous juries are used in all court cases, involving never before convicted
defendants.
Anonymous juries are used in all widely publicized court cases, regardless of identity
of the defendant.
Anonymous juries are used in all widely publicized court cases, involving previously
convicted defendants.

The best answer is A. Anonymous juries are a potential erosion of the presumption of
innocence only because they are now used in cases in which there is a perceived
danger to the jurist, which presupposes a defendant capable of perpetrating a crime. If
anonymous juries were used in each and every case, they would no longer
differentiate between dangerous and harmless defendants.






Alexander Calder was one of the most innovative and original American artists of the
twentieth century. Calder arrived in Paris in 1926 and devoted himself to a innovative
project comprised of animals made out of wire, scraps of cloth, wood, cork, labels,
bits of scrap metal and pieces of rubber that he called the Circus. During his
performances, Calder invented ways to simulate the flight of birds: These are little
bits of white paper, with a hole and slight weight on each one, which flutter down
several variously coiled thin steel wires which I jiggle so that they flutter down like
doves. The Circus was the laboratory of Calders work; in it he experimented with
new formulas and techniques. By 1930, Calder's Circus had developed into one of the
real successes of the Montparnasse art world attracting the attention of such renowned
artists as Fernand Leger and Joan Miro. Encouragement from the upper echelons of
the Parisian art scene undoubtedly led him to try more serious experiments in wire
sculptures.
Calder eventually becoming interested in the movement of objects, some of which he
motorized. In 1933, Calder completed Object with Red Discs, a sculpture he
described as a two-meter rod with a heavy sphere, suspended from the apex of a wire,
giving it a cantilever effect. It had five thin aluminum discs projected at right angels
from five wires, held in position by a spherical counterweight. With this new creation,
the idea of the mobile was born. In creating a work named Constellations in 1943,
Calder explored the plastic possibilities of mobiles; he used small pieces of wood,
which he shaped and sometimes painted. From this point on, Calders ambition
changed focus. He sought more challenging designs. One of Calders objectives was
to display objects in the air, giving the viewer the experience of finding new skies
filled with moving and colored constellations. Calder accomplished this in Acoustic
Ceiling (1954). Calders humor was evident in such works as Le Bougnat (1959) and
The Pagoda (1963). Later, Calder cut fantastic animals from sheet metal, creating La
Vache and Elephant (both 1970) and a mobile entitled Nervous Wreck (1976), which
represents the red skeleton of a fish.
Calder defined volume without mass and incorporated movement and time in art. His
inventions, which redefined certain basic principles of sculpture, have established him
as the most innovative sculptor of the twentieth century.

1. According to the passage, which of the following is an accurate statement about
Object with Red Discs?

It was the first mobile created by Calder.
It was one of the many mobiles without motors created by Calder.
It was one of the many motorized mobiles created by Calder.
It was the first motorized mobile created by Calder.
It was the first of the many mobiles without motors created by Calder.

The best answer is A. According to the passage, Object with Red Discs is Calders
first mobile. It states that Calder became interested in the movement of objects, some
of which he motorized, but there is no information given on whether this particular
sculpture was motorized.

2. According to the passage, all of the following are characteristicof Calders work
EXCEPT

Calder was known to infuse humor into some of his creation
Calder suspended objects from each other
Calder motorized some of his creations
Calder used materials such as metal, cloth, wood, rubber, cork
Calder suspended glass from thin metal wires to create a cantilever effect

The best answer is E. The passage makes no mention of glass as one of the materials
Calder used.


3. The authors attitude toward the mobiles of Alexander Calder is best described as

Hesitance
Detachment
Amusement
Admiration
Indifference

The best answer is D. The author presents only a positive criticism of Calder, stating
that he is the most innovative sculptor of the twentieth century.

4. It can be inferred from the passage that which of the following statement was true
of the Parisian art scene?

The work of Fernand Leger and Joan Miro was influenced by that of Alexander
Calder.
The work of Alexander Calder was influenced by that of Fernand Leger and Joan
Miro.
Fernand Leger and Joan Miro had earned success in the art world before Alexander
Calder.
Alexander Calder had earned success in the art world before Fernand Leger and Joan
Miro.
Calders Circus earned more accolades from the upper echelons of the Parisian art
scene than any other work in its time.

The best answer is C. According to the passage, Calders early work attracting the
attention of such renowned artists as Fernand Leger and Joan Miro. It can be inferred
that Leger and Miro were already famous when Calder was just starting out.










Intuitively, intellectual skills and perceptual-motor skills seem very different because
perceptual-motor skills appear more primitive. Ontogenetically, perceptual-motor
skills develop before intellectual skills, or at least before most intellectual skills are
manifested. Phylogenetically, creatures "high on the evolutionary ladder" are more
obviously capable of intellectual skills than are creatures "lower down

Perceptual-motor skills also seem more closely tied to specific forms of expression.
Being a chess player does not mean one can only play with pieces of a certain size,
that one can only move pieces with one's right hand, and so on. By contrast, being a
violinist means one can play an instrument whose size occupies a fairly narrow range
and that one must play with a rather rigid assignment of functions to effectors
(bowing with the right hand, and fingering with the left). The seeming narrowness of
this perceptual-motorskill expression, contrasted with the seeming openness of
intellectual skill expression, seems to follow from intellectual skills having symbolic
outcomes and perceptual-motor skills having non-symbolic outcomes. Symbolic
outcomes need not be realized in specific ways and can rely on abstract rules. Non-
symbolic outcomes, by contrast, need more specific forms of realization and seem to
depend on restricted associations between stimuli and responses

Another difference between intellectual and perceptual-motor skills is that the two
kinds of skill seem to be represented in different parts of the brain. For example,
structures homologous to the optic tectum, a nucleus located on the dorsal surface of
the midbrain, have a common function in all vertebrates--coordinating visual,
auditory, and somatosensory information relevant to the control of orienting
movements of the eyes, ears, and head. Similarities in structure and function between
these and other brain areas associated with perceptual-motor behavior suggest that
mechanisms for control of perceptual-motor skills are both highly specialized and
conserved across species. In contrast, what distinguishes the human brain from the
brains of other species -- even closely related ones -- is the differential growth of
brain regions most strongly associated with intellectual skills, such as the association
areas of the cerebral cortex.

The contention that these areas serve intellectual functions is supported by a large
body of clinical and experimental literature. Together, these diverse sources of
information suggest that perceptual-motor and intellectual skills depend on distinct
brain circuits

1. The passage is chiefly concerned with

Presenting a new theory and describing a new method to test that theory
Suggesting an alternative to an outdated research method
Demonstrating that perceptual-motor skills are closely tied to specific forms of
expression
Arguing that two seemingly dissimilar skills are more alike than was previously
assumed
Presenting evidence on two dissimilar skills that resolves a contradiction

The best answer is D. The passage presents evidence to back the claims that
intellectual skills and perceptual-motor skills are more similar than was once believed.






2. The author mentions the game of chess in paragraph two primarily in order to

Present an example of an intellectual skill the mastery of which is not closely tied to
specific forms of expression.
Present an example of an intellectual skill the mastery of which is closely tied to
specific forms of expression.
Present an example of a skill that is both an intellectual skill and a perceptual-motor
skill, the mastery of which is closely tied to specific forms of expression.
Present an example of a perceptual-motor skill the mastery of which is not closely tied
to specific forms of expression.
Present an example of a perceptual-motor skill the mastery of which is closely tied to
specific forms of expression.

The best answer is A. Chess is an intellectual skill the mastery of which is not closely
tied to specific forms of expression. In other words, once one has learned to play the
game, one can vary the way one plays, by using ones left hand instead of ones right,
for instance.

3. It can be inferred from the passage that the optic tectum

Functions similarly in animal and in plants
Functions similarly in vertebrates and invertebrates
Is located in a comparable area of the brains of humans and giraffes
Coordinates somatosensory moment in snakes
Has a much more sophisticated structure than the cerebral cortex

The best answer is C. According to the passage, the optic tectum occupies the same
area of the brain in all vertebrates (animals with a spinal column).


4. The passage provides support for which of the following statements

Creatures "high on the evolutionary ladder" are not less capable of perceptual-motor
skills than are creatures "lower down
Playing a violin is a perceptual-motorskill.
The shape and size of the cerebral cortex is what distinguishes the human brain from
the brains of other species
Literature is an intellectual activity that is understood using the cerebral cortex of area
of the brain.
Perceptual-motor and intellectual skills exploit the same brain circuits

The best answer is B. The author uses the violin as an example of a perceptual-motor
skill.


Considerable debate exists in the self-perception literature over the impact of
positively biased self-perceptions on social and psychological functioning. Positively
based self-perceptions are those in which an individual has a more positive opinion of
himself than objective indicators warrant. One view suggests that positive perceptual
biases are characteristic of normal human thought across a variety of domains and
correlate positively with good mental and psychological health. Certain researchers
and clinicians have even proposed that by boosting self-concepts, symptoms of
depression and levels of aggression may be reduced.

Investigators on the other side of the debate maintain that when most positive self-
perceptions are compared to an objective criterion, they appear neither positively
biased nor adaptive. In fact, Baumeister, Smart, and Boden suggest that positively
biased self-concepts may have a dark side. They proposed that it is persons with
very positive self-views who are prone to be aggressive. As a result, building up
individuals' self-perceptions may serve only to increase levels of aggression rather
than curb them.

According to Baumeister et al., not all individuals with positive self-perceptions are
going to be interpersonally aggressive. Rather, individuals who are extremely positive
in their perceptions of themselves and their functioning are proposed to bethe most
likely to become angry and potentially violent. The mechanism that triggers
aggressive behavior by these individuals has been suggested to be negative social
feedback that challenges their positive self-views. Such threats to positive self-esteem
give rise to anger and hostility.

If negative social information is encountered that challenges established positive self-
perceptions, Baumeister et al. propose that individuals must choose to either accept
the feedback and lower their self-perceptions or reject the feedback to maintain their
positive self-views. The chosen reaction then influences their subsequent affective
states and behavioral expressions. By accepting the external appraisals and adjusting
self-perceptions downward, dysphoric feelings and social withdrawal may result.
Conversely, the rejection of the validity of the unfavorable feedback results in
feelings of anger and resentment toward the source of the threat. Dodge and
colleagues demonstrated that children who interpret social cues as threatening direct
their anger and aggression at the peers who gave the negative evaluations. Anger
stemming from the receipt of social criticism is a way to deny the legitimacy of the
negative information. By directing hostile reactions toward the source of the negative
feedback, the influx of disconfirming information may end. Unless individuals react
against the self-esteem threat, they may be compelled to revise their self-concepts
negatively, in line with the information provided. By discounting the negative social
feedback, individuals can protect themselves from dysphoric feelings and maintain
their positively biased self-perceptions, but they may be setting themselves up to
become interpersonally aggressive

Although positively biased self-perceptions may place individuals at risk for negative
social feedback and subsequent increases in aggressive behavior, not all positive self-
concepts are suggested to be harmful. The relationship between positive self-
perceptions and aggression may depend on the degree of perceptual distortion (i.e.,
moderate vs. extreme distortion). Baumeister (1989) and Baumeister et al. (1996)
proposed that an optimal range of moderate bias might exist within which mental
health is encouraged. Maladjustment in psychological and social functioning is
suggested to occur when the degree of bias of self-perceptions shifts from moderate to
extreme levels. Extremely negative and positive perceptual bias would be related to
different but equally harmful difficulties.

1. The primary purpose of this passage is to

Present two explanations of a phenomenon and reconcile the differences between
them
Discuss a plan for investigation of a phenomenon that is not yet fully understood
Challenge the validity of a theory by presenting evidence that the opposite is true in
some cases
Summarize two theories and suggest a third theory that overcomes the problems
encountered in the first two
Present evidence that resolves a contradiction

The best answer is C. The passage challenges the validity of the theory that positively
biased self-perception is closely correlated with normal human thought and good
mental health by showing how a very high self-perception may lead to violent
behavior.


2. According to the passage, which of the following is mentioned as a factor in
determining whether an individual with positively biased self-perception would
actually be likely to perform an act of violence?

The gap between what the individual thinks about himself and how good he really is
The gender of the individual
The anger level of the individuals peers
The individuals ability to ignore dysphoric feelings
The validity of the unfavorable feedback

The best answer is A. It is the distortion between how the person really is and how he
perceives himself that is the most important factor, as opposed to the level of self-
perception itself.


3. The passage discusses the likelihood of violence stemming from which of the
following types of individuals?

An individual with a moderately positive self-perception who receives negative
feedback from his peers
An individual with a highly positive self-perception who receives negative feedback
from his peers
An individual with a highly negative self-perception who receives positive feedback
from his peers
An individual with a highly negative self-perception who receives negative feedback
from his peers
An individual with a dark side who receives negative feedback from his peers

The best answer is B. According to the passage, when negative social information is
encountered by a person with a highly elevated level of self-perception, it may lead to
violence.





4. One function of the fifth paragraph is to

State a conclusion about facts presented in an earlier paragraph
Show the chain of reasoning that led to the conclusions of a specific study
Qualify the extent to which a previously presented theory may apply
Introduce information that confirms an established theory
Provide examples that support a new theory

The best answer is C. Up to the fifth paragraph, the passage discusses the possibility
that individuals with highly positive self-perception are prone to performing violent
acts. Paragraph three qualifies this by stating that not all positive self-concepts are
suggested to be harmful.























Gastrostomy tubes are commonly used to provide nutrition and hydration for patients
unwilling or unable to maintain an adequate oral intake. Among hospitalized patients
aged 65 years or older in the United States, the number undergoing placement of a
gastrostomy tube increased from 61000 in 1988 to 121000 in 1995. In 1990 and
1991, roughly one in every hundred hospitalized patients aged 85 years or older
received a gastrostomy tube.

The short-term mortality rates following gastrostomy placement are high. In a cohort
of more than 7000 American veterans who underwent placement of percutaneous
endoscopic gastrostomy tubes between 1990 and 1992, median survival was 7.5
months and 1-year mortality was 59%. Among Medicare beneficiaries receiving
gastrostomy tubes in 1991, 30-day and 1-year mortality was 24% and 63%,
respectively. Because tube insertion itself is only rarely associated with fatal
complications, the high short-term mortality clearly reflects a substantial underlying
co-morbidity in this population. Most patients receiving gastrostomy tubes have
advanced dementia, other types of severe neurological impairment, cancer, or
advanced failure of other internal organs.

The growing use of tube feeding in a population with limited life expectancy
inevitably raises the following question: Do physicians discuss the benefits and
burdens of tube feeding adequately with patients or surrogate decision-makers before
gastrostomy tubes are inserted? Assessing benefits and burdens is an integral part of
informed decision-making and should precede any elective life-sustaining
intervention. However, anecdotal observations and a recent interview study raise
serious questions about the quality of the informed consent process preceding the
insertion of gastrostomy tubes.

A small body of literature suggests that fully informed patients or their surrogates
might in fact decline permanent tube feeding at a higher-than-expected rate. For
example, in the study by Callahan et al, nearly half of the patients undergoing
gastrostomyplacement (or their surrogates) reported that no alternatives had been
discussed before insertion of the tube. O'Brien and colleagues asked 379 mentally
competent nursing home residents if they would want a gastrostomy tube if they
became unable to eatbecause of permanent brain damage; only 33% expressed a
preference for tube feedings in this circumstance. In an interview study of 121
competent patients with amyotrophic lateral sclerosis, only 28% favored feeding by
gastrostomy



1. The passage is primarily concerned with
(A) the morbidity and mortality rates associated with the use of gastrostomy tubes
(B) the proliferation of the use of gastronomy tubes in patients aged 85 years or older
(C) whether physicians adequately discuss the benefits and burdens of tube feeding
with patients or surrogate decision-makers before gastrostomy tubes are inserted
(D) the growing number of patients undergoing placement of a gastrostomy tube
(E) the complications that often follow the insertion of a percutaneous endoscopic
gastrostomy tube

The best answer is C. The question raised in the passage is not whether to use
gastrostomy tubes and what the dangers of using one might be, rather the passage
examines the issue of informed consent before the insertion of a tube.

2. According to the passage, the high mortality rate following gastrostomy found in
research cited in paragraph two is not necessarily a direct result of the placement of a
gastrostomy tube because
(A) tube insertion itself is only rarely associated with fatal complications
(B) 30-day and 1-year mortality was 24% and 63% respectively in 1991 among
Medicare beneficiaries
(C) only cases in which the percutaneous endoscopic gastrostomy tube was
incorrectly place were examined
(D) all the patients in the studies mentioned suffered from life threatening diseases
(E) most of the patients in the studies mentioned suffered from life threatening
diseases

The best answer is E. In paragraph two it is stated that most subject tested suffered
from one of a number of potentially fatal diseases. Hence, the mortality rate found in
patients that receiving a tube, was not necessarily due to the use of the tube, but to the
disease itself or to some other treatment.

3. It can be inferred from the passage that
(A) there are alternatives to the use of gastrostomy tubes to provide nutrition but not
hydration for patients unwilling or unable to maintain an adequate oral intake.
(B) there are alternatives to the use of gastrostomy tubes to provide hydration but not
nutrition for patients unwilling or unable to maintain an adequate oral intake.
(C) there are alternatives to the use of gastrostomy tubes to provide nutrition and
hydration for patients unwilling or unable to maintain an adequate oral intake.
(D) the alternatives to gastrostomy tubes do not provide adequate nutrition and
hydration
(E) the alternatives to gastrostomy tubes provide more adequate nutrition and
hydration than the gastrostomy tubes

The best answer is C. The passage suggests that if patients were offered alternatives
to gastrostomy tubes, some of them would choose them. We can infer from this that
alternatives exist.

4. The author uses the word only in line ??? [third line from the end] most likely in
order to
(A) highlight the oddity of the decision of the patients
(B) emphasize the relatively low percentage of patients that would opt for a
gastrostomy tube if given the choice
(C) point out the limited value of inserting a gastrostomy tube
(D) distinguish the primary factor in the decision making process of brain damaged
patients
(E) single out a unique merit of gastrostomy tubes for brain damaged patients

The best answer is B. The passage draws attention to the fact that patients and their
surrogates are often not asked if they agree to the use of a gastrostomy tube.
O'Briens study indicates that a relatively small percentage only 33% - stated that
they would want the tube if they were unable to eatbecause of permanent brain
damage.




































A ragtag group of idealistic hackers scattered round the world has created software
and devised a revolutionary method for writing it that poses a direct threat to
Microsoft's revenue. Their programs are already running most of the Internet.
According to a survey by the British consultancy Netcraft, the Web server software
Apache is used by more than half of all websites. Furthermore, software named
Sendmail moves nearly every e-mail message across the Internet, while the BIND
program acts as a traffic cop for most of the global network, directing messages down
the right connections to their final destinations

The proven robustness of these programs is worry enough for Microsoft. To make
matters worse, all of them, and many others, are completely free in two senses: one
does not have to pay for them, and the "source code" in which they are written is
openly available. Additionally, one may modify the programs and even sell the result.

The "open source" movement is Microsoft's worst nightmare: a group of programmers
that it cannot out-compete because its members are not motivated by profit, and which
it cannot buy because it does not exist as a formal company.

In the vanguard of the open source movement is Linux, started in 1991 by a 21-year-
old Finn, Linus Torvalds, who wanted to write a free alternative to Unix, a popular
but costly operating system. Today Linux is used byan estimated 7 million people,
and the number is growing rapidly. One of Linux's advantages is that it runs on almost
any hardware, from multi-processor supercomputers down to Palm Pilots. It is
compact (it can fit on a floppy), highly efficient and very fast

Torvalds did not invent the idea of software that is doubly free but he has stumbled
upon and developed a crucially important Darwinian dynamic. In a commercial
software company, every program is carefully planned, and writing tasks are allotted
unilaterally by the project leader. Linux is different. It is designed as a series of
modules, and anyone can work on any of these interlocking elements. Whether ones
work gets included in the final release depends on the consensus view of how good it
is--natural selection in action. The only reward anyone, even Torvalds, gets for this
work is kudos from fellow hackers. That is enough, it seems, to attract a flow of keen
recruits, typically computer science students or software engineers who code Linux on
the side.

Such purposive anarchy is made possible by the Internet. Trial versions of programs
can be downloaded, and comments sent back to the authors, wherever they are.
Programs frequently evolve on a daily basis. With the help of the Internet, the Linux
model exploitsthe ingenuity of hundreds of programmers and hundreds of thousands
of testers. It is a pool of creativity that Microsoft, with its huge resources, will never
be able to match.

1. The author mentions that a particular bit of software is included into the final
release of Linux only by consensus as an example of
(A) how writing tasks are allotted unilaterally by the project leader
(B) the modularity of Linux
(C) Linuxs unorthodox system of rewards
(D) the careful planning that goes into Linux software
(E) the process of natural selection at work in the field of computer programming
The best answer is E. The author describes the workings of Linux as having a
Darwinian dynamic it works on the principle of natural selection. Only the fittest
software is used.

2. It can be inferred from the passage that if Linux existed as a formal company
(A) it would be used by more than the estimated 7 million people that currently use it
(B) its software would be more costly than Unix
(C) its software would still be less costly than Unix
(D) Microsoft would try to acquire it
(E) it could compete with Microsoft

The best answer is D. According to the passage, Microsoft cannot buy Linux because
it does not exist as a formal company.


3. According to the passage, programmers are motivated to create software for Linux
I. to earn accolades
II. for pecuniary gain
III. to learn how to successfully hack software
(A) I only
(B) II only
(C) I and II only
(D) II and III only
(E) I, II and III

The best answer is A. The author states that the only reward anyone gets for work on
Linux is kudos, or accolades, from fellow hackers.

4. The passage mentions each of the following as factors contributing to the success
of groups creating open source software EXCEPT
(A) the groups use the Internet to gather together hundreds of thousands of testers
(B) the groups do not exist as formal companies
(C) the products are designed to work on Palm Pilots
(D) the groups are not motivated by profit
(E) the programs evolve frequently

The best answer is C. The author mentions Palm Pilots only in relation to the Linux
program. No information is given on whether other open source software is versatile
enough to run on Palm Pilots.


The cyclic rise and fall in population size that has been observed in animal and pre-
modern human populations reflects sequential phases of population growth and
decline. Disregarding the possible selective influences of migration, these cyclic
movements must ultimately be accounted for in terms of the range of variation in
fertility and mortality.

No downward trend in mortality is apparent in any country before the middle of the
eighteenth century, about the same time that population growth began to demonstrate
an exponential curve. The initial period of sustained population growth in nearly
every country for which reliable data are available corresponds with at least two
decisive changes in the death rate. First, the fluctuations in mortality became less
frequent and less drastic. Second, the initial, slow--sometimes imperceptible--decline
inmortality gradually gained momentum and eventually stabilized at relatively low
levels in the twentieth century. Thus, steady increases in life expectancy,
progressively diminishing death rates and more stable and predictable mortality
patterns have accompanied the persistent increments in world population.

In England and Wales, Japan, Ceylon and Chile an exponential pattern of population
growth has accompanied the downward trend in mortality. In England and Wales,
where the transition from high to low vital rates occurred over two centuries, the
exponential growth curve was attenuated only after fertility fell and approached the
low level of mortality; this pattern was less apparent for Japan, where an accelerated
transition occurred over several decades. Although data concerning the relative effects
of mortality and fertility on population growth are incomplete for the early transitional
period, it seems likely that a significant though temporary increase in fertility may
have added momentum to the population explosion set off by steady improvements in
survivorship. The influence of fertility is particularly apparent in the rapid population
growth of currently developing nations that have not yet completed their transitions,
for example, Chile and Ceylon. In most of these developing countries, the death rate
has declined rapidly in recent years, especially since World War II, and the birth rate
has remained high with minor fluctuations. This sudden widening of the demographic
gap has produced unprecedented high rates of population growth.

1. Which of the following best describes the content of the passage?
(A) A discussion of how recently developed methods of monitoring population
growth differ from older methods
(B) A description of some of the factors effecting cyclic changes in population

(C) A chronology of the development of different methods for monitoring population
growth
(D) A proposal for improving the accuracy of current methods to monitor fluctuations
in population
(E) An argument concerning the nature of the exponential pattern of population
growth has accompanied the downward trend in mortality

The best answer is B. (A) is incorrect because no recently developed methods are
mentioned in the passage. (C) is wrong because the information given is not
chronological. (D) and (E) are incorrect because the passage neither proposes nor
argues anything.

2. According to the passage, the fertility rate
(A) in Japan has remained constant over the last several decades
(B) in England and Wales has remained constant over the last two centuries
(C) fell at a similar rate in England, Wales and Japan
(D) fell at a faster rate in Japan than in England and Wales
(E) fell at a slower rate in Japan than in England and Wales

The best answer is D. According to the passage the transition from a high fertility
rate to a low one occurred over two centuries in England and Wales. The fertility fell
in Japan over a period of several decades.

3. According to the passage, population increase in currently developing nations is a
result of
(A) the recent decline in death rate, especially over the last 50 years, and a birth rate
that is consistently high with minor fluctuations.
(B) the recent decline in birth rate, especially over the last 50 years, and a death rate
that is consistently high with minor fluctuations.
(C) the recent decline in death rate, especially over the last 150 years, and a birth rate
that is consistently high with minor fluctuations.
(D) the recent decline in birth rate, especially over the last 150 years, and a death rate
that is consistently high with minor fluctuations.
(E) the recent rise in death rate, especially over the last 50 years, accompanied by a
similar rise in birth rate.

The best answer is A. According to the passage, the death rate in currently
developing nations has declined rapidly since World War II, while the birth rate has
remained high with minor fluctuations.

4. It can be inferred from the passage that the first downward mortality trend
(A) followed great fluctuations in fertility rates
(B) followed great fluctuations in death rates
(C) appeared after World War II
(D) appeared before World War II
(E) appeared around 1750

The best answer is E. According to the passage, no downward trend in mortality was
noticeable before the middle of the eighteenth century.















Researchers criticize the transaction cost economics (TCE) paradigm for over-
generalizing the assumption of opportunism as part of human nature. However, some
suggest that individualists have a higher opportunistic propensity in intra-group
transactions, and collectivists in inter-group transactions. This cultural specification of
opportunism helps TCE to accommodate more effectively some criticisms and more
realistically deal with problems of economic organization in today's global economy.

Transaction cost economics was first proposed by Coase and later popularized by
Williamson. One of its key building blocks is the assumption of opportunism,
because individuals, as Williamson claims, "will not reliably self-enforce promises
but will defect from the letter and spirit of an agreement when it suits their purposes".
Since, evidently, not all economic players are likely to be opportunistic, such a
reliance on the assumption of opportunism has resulted in a torrent of criticisms,
calling it dangerous, unhealthy, bad for practice, and an ethereal hand for
organizational researchers. A primary reason TCE has provoked such a debate is
precisely because it is centered on the assumption of opportunism, which touches on a
fundamental question of human nature.

To further develop this paradigm it is necessary to respond to the criticisms by
clarifying and strengthening this important assumption. To be sure, TCE scholars
never assumed that all (or most) individuals are likely to be opportunistic all (or most
of) the time - a clearly indefensible position. Instead, most individuals are assumed to
be "engaged in business-as-usual, with little or no thought to opportunism, most of the
time". However, TCE suggests that it is the inability to differentiate opportunists,
who may be a minority, from non-opportunists ex ante that necessitates the
assumption of opportunism. TCEs critics, on the other hand, do not suggest that
opportunism does not exist; rather, they caution against an over-reliance on the
opportunism assumption because it may not be realistic to hold this assumption
constant across individuals and organizations around the world. In order to make
further theoretical progress, researchers must tackle the harder and more interesting
issues of what kinds of individuals are likely to be opportunists, under what
circumstances, and to what extent. Such an improved understanding of opportunism is
important in today' s increasingly global economy, in which economic players from
different backgrounds routinely interact with each other.

1. In the passage, the author is primarily concerned with doing which of the
following?
(A) Comparing two different approaches to a problem
(B) Present a criticism of an assumption and suggesting how to clarify it
(C) Describing a problem and proposing a solution
(D) Presenting data and drawing conclusions from the data
(E) Comparing two different analyses of a current situation

The best answer is B. The passage presents criticism that TCEs assumption of
opportunism has provoked and suggests what research should be carried out to resolve
the issue.

2. It can be inferred from the passage that which of the following is true of TCE?
(A) All economic players are likely to be opportunistic
(B) The majority of economic players are opportunistic
(C) The majority of human beings are opportunistic
(D) Not all opportunists are opportunistic under the same circumstances
(E) TCE has provoked such a debate because it is centered on the assumption of
fundamentalism

The best answer is D. According to the passage, TCE advocates claim that
individualists have a higher opportunistic propensity in intra-group transactions, and
collectivists in inter-group transactions. Thus, it can be inferred that not all
opportunists act in an opportunistic fashion under the same circumstances

3. Which of the following would most logically be the topic of the paragraph
immediately following the passage?
(A) Specific ways to determine what kinds of individuals are likely to be
opportunists, under what circumstances, and to what extent
(B) The contributions of TCE to economic theory
(C) Ways in which opportunists take advantage of unsuspecting business men
(D) Nontraditional methods of testing individuals to find out if they are opportunists
(E) The centrality of TSE to the position of opportunists in the business world

The best answer is A. In the last paragraph of the passage it says that in order to make
further theoretical progress researchers must tackle the issue of what kinds of
individuals are likely to be opportunists, under what circumstances, and to what
extent. It is logical that the following paragraph would do so.

4. The quotation in line ??? ["will not reliably self-enforce.] is most probably used
to
(A) counter a position that the author of the passage believes is correct
(B) counter a position that the author of the passage believes is incorrect
(C) elucidate a term
(D) point out a paradox
(E) present a historical maxim

The best answer is C. The quotation is most likely used to explain what is meant by
the term opportunism.












The mystery of cloud formation is as ancient as mankind. Throughout history, man
prayed for rain and feared snow and lightning. Only in the last few decades, was man
able to experiment and answer many questions as to how clouds are formed. Lately, it
has seemed that no almost queries were left in that field. However, a recent look at old
experimental data is threatening to overturn a longstanding theory about how water
droplets freeze within clouds

Suspended water droplets can remain liquid even at temperatures far below the
normal freezing point. Data collected in recent years show that clouds as cold as
37.5C can still contain many liquid droplets of water. Such droplets freeze solid
almost instantly if they bump into each other or are otherwise disturbed Most
scientists have long assumed that a tiny globule of pure, liquid water, when disturbed,
begins to freeze around an icy seed that suddenly forms inside it. According to this
scenario, the time needed to freeze a given volume of water dispersed into a fine mist
is independent of the size of the individual droplets because the formation of a seed
particle is a chance event
However, the results of recent laboratory experiments, when combined with
information gathered from tests conducted as many as 30 years ago, do not support
this scenario. Together, the data indicate that the time needed to freeze a given
volume of liquid water varies drastically according to droplet size. This extreme
variation makes sense if freezing begins at the surface of the drops, not at the cores.
Dividing a given volume of water into a large number of small droplets yields more
total surface area than if the volume is split into a small number of large drops. The
freezing rate would then depend on the surface area

The laws of thermodynamics also argue against ice nuclei forming inside liquid
droplets. When water molecules begin to assemble into ice crystals, they release large
amounts of latent heat. If that process had occurred in the center of a liquid droplet,
the heat would have remained trapped within the globule, slowing the freezing
process. But if crystallization begins at the droplet's surface, latent heat can more
easily transfer to the surrounding air. In this case, the droplets are so cold that heat
released internally as crystallization proceeds probably would not melt the developing
ice

Familiar as clouds are, the behavior of their constituent droplets remains only partly
understood. Lightning, rainfall, and other meteorological phenomena vary with the
ratio of water droplets and ice particles in clouds. Linking freezing rates of clouds to
those atmospheric and other climate processes is one of the most unreliable areas in
current climate simulations.

1. According to the passage, if a given volume of water is divided into a large
number of small droplets, as opposed to a smaller number or large droplets, it will
freeze quicker. This indicates that
(A) a given volume of water yields more droplets when below freezing
(B) a given volume of water yields more droplets when above freezing
(C) freezing begins simultaneously both at the cores and at the surface of the drops
(D) freezing begins not at the surface of the drops, but at the cores
(E) freezing begins not at the cores of the drops, but at the surface

The best answer is E. If the relatively large amount of surface area provided by small
droplets influences the time it takes to freeze, then freezing must begin on the surface
of the droplets.

2. According to the passage, all of the following are characteristic of cloud droplets
EXCEPT
(A) they periodically bump into each other
(B) the time needed to freeze a given volume of water dispersed into a fine mist is
dependent on the size of the individual droplets
(C) can remain liquid even at temperatures far below the normal freezing point
(D) they are only partly understood by scientists
(E) they appear in the same amount in clouds involved in various meteorological
phenomena

The best answer is E. According to the passage, the ratio of water droplets and ice
particles in clouds varies in lightning, rainfall, and other meteorological phenomena

3. The passage suggests that many questions in the field of meteorology
(A) have been answered
(B) have no answers
(C) have yet to be answered
(D) have been answered incorrectly
(E) are uninteresting to scientists today
The best answer is A. The author states that it has lately seemed that no almost
queries were left in meteorology.

4. The primary purpose of the passage is to
(A) present several explanations for a well-known fact
(B) argue in favor of a long-standing theory
(C) discuss the implications of a new research finding
(D) present new evidence that overturns a long-standing theory
(E) question the methodology used in a study.

The best answer is D. The passage presents new evidence on how water droplets
freeze which overturns a long-standing theory that claims that droplets freeze from
their center.











To unravel the extent to which Shakespeare is accorded cultural capital in India, two
examples from popular culture will be examined - one from an Air India
advertisement and another from a Hollywood film in the making that casts
Bollywood star Hrithik Roshan as an Indian king in Rajasthan, looking to avenge
the murder of his father.

The Air India advertisement, found in a 1975 American magazine, features
Shakespeare with a beautiful Indian woman clad in a sari. Focused from shoulders
up, the black and white image reproduces a typical studio shot of a married couple.
The top of the ad reads, We go out of our way to please you. We also go to London
every day." This is followed by the contact reference to Air India. Presented in the
image of a couple, the ad naturalizes the union of the colonized Indian woman and
Shakespeare (the most authoritative representative of the culture of the colonizer) and
thrusts them into the global realm of tourism, art, and commerce to symbolize the
union of East and West, with Shakespeare representing the means of transcending the
distance between India and England. To make it more enticing, the slogan that
follows the main caption of the ad is also somewhat sexualized: "We (Air India) work
all day to make your night with us a dream." The advertisement creates aspectacle
that reproduces Shakespeare in the image of the authoritative imperial traveler to the
exotic and mysterious land, India, represented through the image of the exotic Indian
woman. In so doing, the advertisement at once obscures the gendered, racial, and
colonial dynamics that complicate such an image and legitimizes the discourse around
Shakespeare as a harbinger of cultural authority

My second example is a Hollywood film in the making that plans to cast Indian
Bollywood star Hrithik Roshan in the role of a Rajput prince who avenges his father's
murder in a "chilling thriller." To generate publicity for the film, the filmmakers
proposed to market Roshan as an "Indian Hamlet." While the producer believes that
Hrithik is the ideal candidate for the role and places him "somewhere between Tom
Cruise and Antonio Banderas," he recognizes that Hrithik is a new name in
Hollywood. Therefore, he says, "we will have to sell him through the two known
names--Hamlet and Shakespeare". In doing so, the Hollywood producer assumes the
far-reaching influence of and appreciation for Shakespeare. At the same time, as a
film that will be shot in India with some scenes in England, it conjures up the image
of exoticism that continues to seize the West's imagination.

Such images not only keep alive Shakespeares insidious influence in India, they
further perpetuate the discourse about the bard's cultural superiority, which continues
throughgovernment-sponsored agencies, the continuing presence of Shakespeare
studies in education, and through theatre groups and touring companies performing
locally and visiting from abroad.

1. According to the passage, Hrithik has to be marketed as Indias Hamlet because
(A) his name is unknown to American audiences
(B) American audiences confuse him with Tom Cruise and Antonio Banderas
(C) the movies Hollywood producer assumes the far-reaching influence of and
appreciation for Shakespeare
(D) the film will be shot in India and England
(E) Hrithik is the ideal candidate for the role
The best answer is A. The author states that since Hrithik is a new name to American
audiences he will have to be sold through two familiar names, Hamlet and
Shakespeare.

2. The authors attitude toward the cultural capital accorded to Shakespeare in India
is best described as one of
(A) indifference
(B) hesitance
(C) disapproval
(D) amusement
(E) neutrality

The best answer is C. In the last paragraph, the author refers to Shakespeares
influence as insidious.

3. In can be inferred from the passage that more people in the U.S. are likely to pay to
see the Hollywood movie mentioned in the passage if the actors are
(A) Indian
(B) British
(C) Classically trained
(D) familiar to the audience
(E) excellent at portraying their roles

The best answer is D. No information is given in the passage on how the factors
mentioned in choices A, B, C, and E might influence a movie-goer.

4. In the last paragraph, the author is primarily concerned with
(A) correcting an error occurring in one of the works under review
(B) citing evidence to support a view of Shakespeare in India
(C) drawing conclusions on the basis of evidence presented in the first three
paragraphs
(D) summarizing the arguments about Shakespeare presented in the first three
paragraphs
(E) refuting the view of Shakespeares influence in India presented in the previous
paragraph

The best answer is C. In the last paragraph, the author draws conclusions on the
effects of the examples presented earlier in the passage.









The relationship between science and religion, and even the one between skepticism
and religion, is warming up. At least, that is the feeling one gets from a cursory look
at recent happenings, including the proliferation of books and articles in popular
magazines about science "finding" God.

Thus, the time is ripe for a skeptical analysis of the subject, which seems muddled by
two basic sources of confusion: the need to separate logical/philosophical arguments
from those that are either pragmatic or concern freedom of speech; the need to
acknowledge that there are many more possible positions on the science and religion
question than are usually considered, and that a thorough understanding of the whole
gamut is necessary to make any progress.

The relationship between science and religion is a legitimate area of philosophical
inquiry that must be informed by both theology and science Discussions about
science and religion, especially in the United States, carry practical consequences that
do not affect both in an equal manner. Discussing science and religion has
repercussions on the cherished value of freedom of speech for scientists, skeptics, and
religionists

Attacks on religion are considered politically incorrect--the remarks by Minnesota
Governor Jesse Ventura resulted in his popularity dropping 28 percent overnight.
Scientists are especially aware of the fact that their research funding depends almost
entirely on public financing through various federal agencies such as the National
Science Foundation and the National Institutes of Health. Since federal funding is
controlled by politicians, who in turn have a tendency to respond to every nuance of
their constituency as gauged by the latest poll, it follows that no matter what your
opinion as a scientist on matters of the spirit, it is wiser to stick to your job and avoid
upsettingyour benefactor

This is all the more so because of two other things we know about scientists: the
overwhelming majority of them do not believe in a personal God (about 60% of
general scientists and a staggering 93% of top scientists), and the reason they become
scientists is to pursue questions for which science is a particularly good tool. Most of
these questions are rather more mundane than the existence of God

The result of this odd mix is that while most prominent scientists do not believe in a
personal God because of their understanding of science and of its implications, they
must come out in public with conciliatory statements to the effect that there is no
possible contradiction between the two.
1. The author mentions Governor Jesse Ventura (lines ???) most likely in order to
(A) prove that the Governor does not believe in a personal God
(B) show that the Governor believes in a personal God and therefore does not believe
that various federal agencies such as the National Science Foundation and the
National Institutes of Health should back scientific enquiry
(C) show that the public is unlikely to finance a politician who denounces religion
publicly
(D) show that the public is unlikely to support a public figure who denounces religion
publicly
(E) show that the public is likely to back a public figure who denounces religion
publicly

The best answer is D. The author uses the governors case to show that if a public
figure speaks out against religion that person would likely lose the support of the
public

2. It can be inferred from the passage that the author believes that the questions asked
by scientists are
(A) more ordinary than questions about the existence of God
(B) more extraordinary than questions about the existence of God
(C) more popular than those asked by theologians
(D) more popular than those asked by politicians
(E) less extraordinary than those asked by politicians

The best answer is A. The author states that science delves into questions that are
more mundane than the existence of God.

3. The author implies that which of the following will occur if a scientist publicly
declares that he does not believe in a personal God?
(A) That scientist will likely lose his job.
(B) That scientist will likely lose all support.
(C) That scientist will likely lose federal support.
(D) That scientist will likely lose all funding.
(E) That scientist will likely lose federal funding.
The best answer is E. According to the passage, a scientist that speaks out against a
belief in God would likely lose federal funding because that funding is decided upon
by politicians who do not want to lose the support of their voters.

4. The passage as a whole can be characterized as which of the following?
(A) A description of an attitudinal change
(B) A discussion of an analytical defect
(C) A look at the interrelationship of two fields of inquiry
(D) an argument in favor of revising a view
(E) an evaluation of a scholarly study

The best answer is C. The passage examines the relationship between science and
religion.











The Technology-Related Assistance for Individuals with Disabilities Act of 1988
defines assistive technology devices as any item, piece of equipment, or product
system, whether acquired commercially, modified, or customized, that is used to
increase, maintain, or improve functional capabilities of individuals with disabilities.
The promise offered by this law is that a student with a disability will be offered any
assistive devices and services necessary to enhance his or her educational experience.

Although there is general agreement that technology is educationally beneficial, the
research results have not strengthened the case for its impact on teaching children
with disabilities. However, Edwards not only encourages an expanded use of
technology in education, but also offer several reasons why its continued use is
justified. For example, he believes that technologically based methods have promoted
several motivational strategies such as gaining learner attention, getting learners to
create their own technologically based products, and empowering learners to take
control of their own learning. He also contends that technology can facilitate unique
learning environments, make traditional learning environments more powerful and
more effective by linking learners to information sources, help learners visualize
problems and solutions, and connect learners to learning tools

While most agree that technology has obvious appeal for improving educational
outcomes for students, teachers must ensure that it accommodates the specialized
needs of students with disabilities. The diversified needs of students with various
disabilities (e.g. behaviordisorders, mental retardation, blind or vision impairment,
deafness and so on) require different technological resources and applications. For
example, assistive technology devices have been designed to bypass the need to type
for learners with physical disabilities that effect their hands

One should consider the following factors when selecting and using assistive devices.
The user should ensure that the assistive device can be integrated into the student's
instructional program, can endure rapid technological changes and, if warranted,can
be upgraded, and can be easily maintained and that technical support, repair or
maintenance is readily available. Additionally, school districts should ensure that
teachers receive the necessary training and support for employing assistive devices
and services so that teachers can provide students with the necessary motivation to
use assistive devices
1. According to the passage, research on the use of technology in education
(A) has not strengthened the contention that it should be used to educate
children with disabilities
(B) has not strengthened the contention that it should be used to educate
children without disabilities
(C) has not strengthened the contention that it should be used to educate deaf
and hearing impaired children
(D) has strengthened the contention that it should be used to educate deaf and
hearing impaired children
(E) has strengthened the contention that it should be used to educate physically
handicapped children

The best answer is A. According to the author, although there is general
agreement that technology is educationally beneficial, the research results have
not strengthened the case for its impact on teaching children with disabilities.

2. According to Edwards which of the following is true about technologically
bases methods
(A) Technologically based methods make learning more enjoyable, making it
possible to absorb more information.
(B) Technologically based methods help disabled learners overcome their
shyness by connecting them virtually with other non-disabled students
(C) Technologically based methods shorten the learning curve for disabled
students
(D) Technologically based methods empower learners to take control of their
own learning.
(E) Technologically based methods circumvent to need for disabled student to
get information for libraries that may be inaccessible.

The best answer is D. In paragraph two, the author mentions students taking
control of their own lives as one of the advantages of technologically based
methods cited by Edwards.

3. According to the passage, all of the following can be considered assistive
technology as defined by the Technology-Related Assistance for Individuals
with Disabilities Act of 1988 EXCEPT
(A) a product system used to improve functional capabilities of individuals with
disabilities
(B) a piece of equipment that has been modified to improve functional
capabilities of individuals with disabilities
(C) a piece of equipment that has been customized to replace the use of a limb
in an individual with physical disabilities
(D) a piece of equipment that has been modified to maintain functional
capabilities of individuals with disabilities
(E) a commercially acquired item used to increase functional capabilities of
individuals with disabilities

The best answer is C. The passage mentions increasing, maintaining, improving,
but not replacing functional capabilities of individuals with disabilities.


4. According to the passage, the value of a particular type of assistive
technology depends on its ability to be which of the following?
I Upgraded
II Easily maintained
III Link learners to information sources

(A) I only
(B) II only
(C) I and II only
(D) I and III
(E) I, II, and III

The best answer is C. In paragraph four, the author lists factors to consider
before purchasing assistive software. I and II are included in the list.
Economists have defined four stages societies pass through based on demographics,
economics, agricultural productivity, and technological advancements. The first stage
is a traditional society, characterized by low population density and low economic and
population growth. In this society, which may be a country or a region, high birthrates
match high death rates, while primitive technology contributes to low income and low
living standards.

The second stage is a developing stage, occurring when a society's technological
advancements result in sustainable agricultural production and plant and animal
domestication. The result is a more plentiful food supply, which helps increase
population growth mainly by slowing death rates. When population and food
production growth are combined with industrialization and urbanization at this stage,
the result is environmental exploitation and degradation present in many developing
societies

Increases in agricultural productivity and production bring economic surpluses that
allow growth in capital and per capita income and the third stage, the developed
society, is born. Birthrates fall faster than death rates as the roles of women change
and developments in birth-control methods allow adults to choose the number of
children they desire

Finally, stage four, the mature society, sees notable technological change evolve
beyond agriculture (particularly in medicine and public health), making death rates
decline further. While many technological breakthroughs in developing societies
come from innovative laypersons, breakthroughs in developed societies tend to
require scarce, highly trained, experienced, and costly technicians and scientists. By
the time the mature society develops, the most readily accessible raw materials have
been exploited. Obsolescence of current technology requires investment in
maintenance rather than in new technologies. Increase in productivity of service
activities, which grow in importance, becomes more difficult than increases in
agricultural and manufacturing productivity. Some developed regions choose to
sacrifice some economic growth for equity. Thus, while productivity and income
continue to rise, the rate of these increases slows.
Rapid agricultural productivity gains continue in developed societies as investments
in education and science made in the development stage produce long-term payoffs
and as urbanization and industrialization lead to an exodus of agricultural labor. At
the same time, slowing rates of income growth and population growth slow down the
growth in demand for food. Food self-sufficiency increases in some countries after
falling in the development stage. However, agricultural trade typically grows, as more
affluent consumers demand a variety of foods from around the world

Many developed countries have recently entered or will soon enter the fourth stage--
the mature society--the future society of the world's inhabitants. It is a long-held view
that global population growth will more or less stabilize; recent evidence, however
presents a strong case for negative global population growth as the seminal attribute
of the mature society

1. The primary purpose of the passage is to
(A) discuss a plan for investigation of a process
(B) summarize a theory and suggest a revision that overcomes a problem with
the theory
(C) describe a course of development
(D) present a mechanism that causes change and development
(E) challenge the validity of a theory by exposing inconsistencies

The best answer is C. The passage describes the development of societies from
stage one to stage four.

2. It can be inferred from the passage that the birth rates in a third stage
society is
(A) the same as that of a fourth stage society
(B) lower than that of a fourth stage society
(C) the same as that of a second stage society
(D) lower than that of a second stage society
(E) faster than that of a second stage society

The best answer is D. According to the passage, the birth rate in a third stage
society falls faster than the death rate, which indicates that the rate is falling.
Hence, the birth rate must be lower than that of a second stage society.


3. According to the passage, the categorization of societies into four stages
outlined in the passage has been created by
(A) industrialists
(B) economists
(C) politicians
(D) the United Nations
(E) sociologists

The best answer is B. The first sentence of the passage starts: Economists have
defined four stages ..

4. According to recent evidence, negative global population growth is considered
(A) a superfluous attribute of a mature society
(B) a common aspiration of a mature society
(C) the most important determining attribute of the mature society
(D) the most important determining attribute of the developed society
(E) a common myth of a developing society

The best answer is C. The author states that negative global population is the
seminal attribute of the mature society








According to the United Nations, the total fertility rate - the number of children a
woman may be expected to bear during her lifetime - has fallen in every region of the
world since 1950. From an average of nearly six children per woman in the 1950s,
total fertility rate fell to three children in Latin America, 3.4 in India, and 3.5 in other
parts of Asia by theearly to mid-1990s. The only major exception to this sustained
downtrend is in North America, where the recent increase in total fertility rate appears
to be a transitory phenomenon associated with immigration and a large number of
baby-boom women deciding to have children relatively late in their lives

As significant as declining total fertility rates worldwide is the fact that, from 1990 to
1995, the rates in Europe, China, and North America were below the 2.1 average
children per woman needed to sustain population worldwide over the long run. The
United Nations' medium population projection of 2.1 children after 2040 is widely
used as a demographic forecast, but it unrealistically assumes that this rate will be the
same in both developed and developing countries. Many researchers, such as
Wolfgang Lutz of the International Institute for Applied Systems Analysis, do not
support the UN assertion that fertility would increase to replacement level in
developed countries. Lutz and others cite evidence pointing toward low fertility,
noting contraception, declining marriage rates, high divorce rates, increasing
independence and career orientation of women, materialism, and consumerism.

These factors, together with increasing demands and personal expectations for
attention, time, and also money to be given to children, are likely to result in fewer
couples having more than one or two children and an increasing number of childless
women,"Lutz and his colleagues write in The Future Population of the World. The
United Nations has a second scenario - the low/medium scenario - that presumes
fertility averaging 1.9 children per woman for all regions by 2025. This scenario may
be as unrealistic as the medium population scenario. The low/medium scenario may
underestimate future total fertility rates in developing countries, just as the UN
medium scenario may overestimate future total fertility rate in developed countries

The low/medium scenario projects a peak world population of 7.9 billion people in
2050, declining to 6.4 billion by 2150. The medium scenario projects a peak world
population of just less than 11 billion by approximately 2200. Most other projections,
however, predict peak global population in less than a century, followed by negative
population growth
These 1998 UN population figures were revised in 2000, and the new estimates,
though tentative, indicate population trends even lower than the 1998 predictions. The
low/medium scenario is comparable to what the United Nations now calls the low
variant forworldwide populations. Given the low variant, the United Nations predicts
that world population in 2050 will be 7.8 billion--slightly less than the 7.9 billion
projected under the low/medium scenario. It is predicted that the United Nations will
continue to revise population trends downward and that negative population growth
will occur even sooner than 2150. However, because the 2000 population data remain
tentative and do not extend to 2150, it is preferable to continue to rely on the 1998
data.

1. Which of the following statements can be inferred from the passage?

(A) immigrates to the United States give birth to fewer children than they would
have, had they remained in their native countries
(B) immigrates to the United States give birth to more children than they would
have, had they remained in their native countries
(C) more immigrates to the United States give birth to fewer children than they
would have, had they remained in their native countries
(D) less immigrates to the United States give birth to more children than they
would have, had they remained in their native countries
(E) immigrates to the United States give birth to fewer children, on average,
than native born American woman

The best answer is E. Since elevated fertility rate can be partly accounted for by
immigration, it can be inferred that the immigrants give birth to more children
than do native Americans, on average.

2. It can be inferred from the passage that
(A) the birth rate in South-East Asia has risen since the 1950s
(B) the birth rate in Africa has declined since the 1950s
(C) the birth rate in North America has declined since the 1950s
(D) the birth rate in North America is higher than the death rate
(E) the birth rate in North America is lower than the death rate

The best answer is B. According to the passage the fertility rate has declined in
every region of the world except North America, therefore it can be inferred that
the fertility rate dropped in Africa.

3. According to predictions made by Wolfgang Lutz, which of the following is
likely to occur?
(A) More women in future will remain childless than do so today.
(B) More woman will put off having children until their thirties than do so today
(C) More woman will give birth one or two babies than do today.
(D) Less women will choose not to have children.
(E) The number of children a couple has will be determined more and more
often by the female partner.
The best answer is A. According to the passage, Wolfgang Lutz predicts that
various factors will lead to an increasing number of childless women.

4. According to the United Nations projected demographics, the low/medium
forecast predicts
(A) a peak in approximately 2050 while the medium forecast predicts a peak in
approximately 2200
(B) a peak in approximately 2050 while the medium forecast predicts a peak in
approximately 2150
(C) a peak in approximately 2150 while the medium forecast predicts a peak in
approximately 2050
(D) a peak in approximately 2150 while the medium forecast predicts a peak in
approximately 2200
(E) a peak in approximately 2200 while the medium forecast predicts a peak in
approximately 2150

The best answer is A. The low/medium forecast predicts that the population will
reach its highest point - 7.9 billion - in approximately 2050. The medium
forecast predicts that the population will reach its highest point - 11 billion - by
approximately 2200.

The racial identity of South Asians has long been a subject of some controversy in the
United States. In the early years of the twentieth century, when whiteness, or African
ancestry, was a prerequisite for naturalization, American courts vacillated on the
question of whether Asian Indians were white or not. In contrast to Mexicans and
Armenians, who were deemed white for the purposes of citizenship acquisition, and
Japanese, Chinese, and Filipino applicants who were not, the verdict on the racial
classification of Indians changed from case to case. American uncertainty over South
Asian racial identity has also been mirrored in the Census Bureau's frequent changes
in its classification of this group. Over the course of the last century,respondents of
South Asian origin have been classified variously as `Hindu', `White', `Other', and
`Asian'.

South Asian newcomers are not alone, however, in confronting an American racial
landscape that at first seems to have no clear place for them. Not only does the
diversity of the United States' contemporary immigrant pool ensure a steady influx of
people who do not fit easily into the traditional black/white dichotomy, but in the past
as well, immigrants tested, stretched and molded the nation's conceptions of racial
categories. As Ignatiev has shown, Irish immigrants were not considered white until
well after their arrival in the United States, and this was true of other European groups
as well. Similarly, Chinese, Japanese, Korean, and Filipino Americans were not
always considered to constitute a pan-ethnic Asian race.

But unlike the Irish who have already become white, or the Chinese and Japanese
who are now Asian, the racial classification of South Asians in the United States is
still in flux. Although they now seem firmly ensconced in the census `Asian' category,
thisis a recent development and one that came about only after considerable debate.
Moreover, several writers have described an uneasy alliance between South Asians
and East Asians under the pan-ethnic `Asian' rubric. Finally, other Americans seem
unsure as to the racial status of these immigrants. F. James Davis finds evidence that
some blacks consider Indians to be black as well, and Rosemary Marangoly George
reports a widespread concern among Indian Americans in California over being taken
for Mexican or black. More broadly, Nazli Kibria maintains that South Asians are
seen as `ambiguous non-whites' in the United States

Defined as `the socio-historical process by which racial categories are created,
inhabited, transformed, and destroyed', racial formation is both a macro-level process
and the culmination of myriad individual encounters. Given their inchoate racial
status, South Asian Americans may offer unusual insight into this process of racial
formation.
1. The main topic of the passage is
(A) the socio-historical process by which racial categories are created
(B) the racial status of ambiguous non-whites in the United States
(C) census bureau classifications in the United States
(D) the process of racial formation as exemplified by the case of the South
Asians in the United States
(E) the racial identity of South Asians in the United States

The best answer is E. Choices A and D are too general. The passage mentions
ambiguous non-whites and census bureau classifications only as they relate to
the identity of South Asians in the U.S.

2. The author mentions the Irish most likely in order to
(A) prove how flexible racial categories can be over time
(B) suggest that they may not really be white
(C) counter the claim that all non-Africans are white
(D) suggest that South Asians follow their example
(E) differentiate between them and immigrants from China, Japan, Korea, and
the Philippines

The best answer is A. Racial categories are considered by most to be fixed. The
author gives the example of the Irish to prove that they can be flexible.


3. The information in the passage suggests that in the early twentieth century, it
would have been difficult for a person from which of the following ethnic
groups to obtain citizenship?
(A) A Briton
(B) A Filipino
(C) A Japanese
(D) An Armenian
(E) An American of African descent
The best answer is B. Britons and Armenians were considered white and so
would be granted citizenship. Citizenship was also granted to people of African
descent.

4. According to the author, which of the following is true of racial formation?
(A) It is an irreversible process.
(B) It is a social-historical process.
(C) It offers unusual insight into South Asians.
(D) It offers unusual insight into the human race.
(E) It is a racist practice.

The best answer is B. In paragraph four they author defines racial formation as
the socio-historical process by which racial categories are created.















In parallel with advances in drug therapy, anesthesia, and surgery, there has been a
shift in bioethics from a paternalistic ethic governed by doctors to one based on the
patient's autonomy and integrity. The notion of consent to medical procedures and
treatment is a reflection of this, and in many countries, this consent is now established
in law.

The terms "informed" and "patient's consent" were perhaps first coupled in the 1957
case of Salgo versus Leland Stanford Jr. University Board of Trustees. The plaintiff,
paralyzed after myelography, had not been informed by his doctor that paralysis was a
possible risk of this procedure. He won although the doctor had committed no mistake
because the doctor failed in his duty of disclosure. The court found that if the patient
had been properly informed he would have refused myelography.

From a legal perspective, any consent, if it is to be valid, has to meet three
independent preconditions, all of which have to be taken into account simultaneously.
Consent may be deemed invalid if it has been obtained by deception or coercion; if it
does not comply with formal procedures; or if the person lacks the capacity to consent
by virtue of mental illness. The weaker a patient's personal competence, the more
stringent the procedural considerations must be. This is to avoid coercion or
exploitation, and to ensure that the disclosure of information has taken into account
the patient's capacity for understanding and evaluating the situation.

Equally, certain radical medical procedures call for stringent requirements to be met
on personal and procedural competence. Such preconditions may be specifically
formulated, as in Norway's abortion, sterilization, and transplantation legislation.
These preconditions reflect a general principle of international health law, illustrated
by the 1973 US case of Kaimowitz vs Michigan Department of Mental Health. Here
the court found that even though a difficult psychiatric patient possessed the
competence required to consent to ordinary surgical procedures, and even to
"accepted neurosurgical procedures", his competence would not be sufficient for him
to consent to experimental neurosurgical procedures characterized as "dangerous,
intrusive, irreversible, and of uncertain benefit to the patient and the society".

The requirement for informed and voluntary consent is not always applicable to non-
invasive medical procedures. In most cases, staff should be able to decide on the
necessary procedures without having the patient's express consent, after having
provided information to the patient. They must, however, respect a patient's rejection
of any specific examinations or treatments. When the treatment offered is invasive,
the doctor will have the responsibility for providing the patient with all necessary
information--about the risks and the alternative treatments and their probable
consequences.


1. According to the passage, when a treatment is invasive,

(A) difficult psychiatric patients cannot consent to ordinary surgical procedures,
and even to "accepted neurosurgical procedures
(B) difficult psychiatric patients must have a family member give written
consent for any surgical procedures.
(C) the patients express consent is necessary and it is the doctors
responsibility to obtain it from the patient after he or she has received all
the necessary information
(D) the patients express consent is necessary and it is the staffs responsibility
to obtain it from the patient after he or she has received all the necessary
information
(E) the patients express consent is if the patient has received all the necessary
information about the risks and the alternative treatments and their
probable consequences.

The best answer is C. Consent is necessary for invasive procedures, and it is the
doctors responsibility to obtain it, not the staffs.

2. Which of the following, most accurately states the purpose of the passage?
(A) To compare two different approaches to the question of consent.
(B) To summarize two court cases regarding the question of informed consent.
(C) To argue for a particular interpretation of the term "informed consent.
(D) To cite examples of how the notion of informed consent has been abused by
medical staff working with psychiatric patients.
(E) To discuss the notion of informed consent, its history and some variations on
how the term is applied.

The best answer is E. The passage deals with various aspects of informed
consent, including the history of the term and some special consideration to how
it is applied.



3. It can be inferred from the passage that prior to 1957
(A) doctors were allowed to continue performing the potentially dangerous
myelography procedure
(B) doctors were allowed to continue performing the potentially dangerous
myelography procedure without the patients consent.
(C) doctors were not allowed to neglect informing their staff of the dangers of a
medical procedure and the alternatives that exist.
(D) doctors were not required by law to inform their patients of the dangers of
a medical procedure and the alternatives that exist.
(E) doctors were required by law to inform their patients of the dangers of a
medical procedure and the alternatives that exist.

The best answer is D. According to the passage, this case laid the precedent for
informed consent, therefore, it can be inferred that consent was not required
prior to the case.

4. Which of the following can be inferred from the passage about Norways
abortion legislation?

(A) Abortion legislation in Norway requires that the patient sign a special
consent form that is different from the general consent form required for
other medical procedures.
(B) Abortion legislation in Norway does not require that the patient sign a
consent form because the patient is assumed to have personal and
procedural competence.
(C) Abortion legislation in Norway does require the patient to sign a consent
form only when the attending physician believes the patient lacks personal
and procedural competence.
(D) Abortion legislation in Norway require the patient to sign a consent form
only even when the attending physician believes the patient lacks personal
and procedural competence.
(E) Legislation in Norway requires the patient to sign a consent form only when
a procedure is not intrusive so the law does not deal with abortion.

The best answer is A. According to the passage, the preconditions for abortion
and other procedures are specially formulated.


























Resuscitation after severe hemorrhage is a very delicate process that involves many
biological procedures that need to be carried out within a very limited time window. To
successfully revive a victim of severe exsanguination, one must work quickly within the
"golden hour" time limit. In this time, there are various traumas that must be avoided;
cardiac arrest, hypotension, hemorrhagic shock, and hypothermia to name a few.

The resuscitation process involves the restoration of normotension (normative blood
pressure) while avoiding cardiac arrest and hemorrhagic shock. Even when normotension
has been restored, hemorrhagic shock, which can cause organ failure, can be a concern.
There are three types of hemorrhagic shock: compensated hemorrhagic shock;
uncompensated hemorrhagic shock, which is reversible; and irreversible hemorrhagic
shock. The most common method of resuscitation makes use of lactated Ringer's solution
to compensate for lost blood volume by causing the cells to swell, which in turn restores
normotension. Though this works fairly well, it is not the optimal treatment.

Half of the deaths that occur annually are due to acute illness or injury, and are associated
with circulatory failure or shock. Some of these deaths could be avoided by proper
monitoring. Present technology involves monitoring early in the temporal course of an
acute illness to observe the cardiac index, oxygen delivery and oxygen consumption. In
future, a possibility for a very effective non-invasive monitoring device, would be one
which could provide the following cardiac output readings: pulse oximetry for estimating
arterial hemoglobin oxygen saturations, a reflection of pulmonary function;
transcutaneous
oxygen and CO2 tensions, reflections of tissue perfusion; and noninvasive blood pressure
readings. Non-invasive systems are far more effective than invasive systems because they
provide a constant display of the data and can be used at any location, whether in the
hospital or in the field. In future, if systems like these are perfected, shock may be easily
intercepted and avoided, thus resulting in a significant number of saved lives.

Future studies on resuscitation should be centered around shock, prevention of cardiac
arrest as well as on increasing oxygen transport and increasing blood volume. The validity
of blood pressure as a measure of organ viability and optimum possibility of resuscitation
should be investigated as well. That said, resuscitation research should center mainly on
developing procedures that can do all of these quickly and efficiently so that resuscitation
will save lives everywhere - from hospital to battlefield.

1. In this passage, the author is primarily interested in
(A) describing the resuscitation process and suggesting strategies for improving
it.
(B) advocating particular strategies for future efforts to improve the
resuscitation process
(C) suggesting an alternative to an outdated research method
(D) distinguishing between different resuscitation strategies
(E) evaluating the effectiveness of resuscitation

The best answer is A. The passage both describes the resuscitation process and
suggests how it may be improved in future.

2. The author mentions which of the following as a possible consequence of
severe hemorrhage?
(A) hypertension
(B) normotension
(C) increased blood volume
(D) hemorrhagic shock
(E) lactated Ringers solution

The best answer is D. Paragraph two of the passage discusses hemorrhagic
shock, which, according to the passage, may occur after a severe hemorrhage
even if normotension has been restored.


3. According to the passage, which of the following statements is true about
monitoring technology?
(A) The present technology observes CO
2
delivery and consumption, while
future technology might provide readings on pulse oximetry and
transcutaneous oxygen and CO
2
tensions.
(B) The present technology observes oxygen delivery and consumption, while
future technology might provide readings on pulse oximetry and
transcutaneous oxygen and CO
2
tensions.
(C) The present technology observes oxygen delivery and consumption, while
future technology might provide readings on pulse oximetry and
transcutaneous oxygen and carbon monoxide tensions.
(D) The present technology observes oxygen delivery and consumption, while
future technology might provide readings on pulse oximetry and
transcutaneous oxygen and CO
2
saturations.
(E) The present technology regulates oxygen delivery and consumption, while
future technology might evaluate pulse oximetry, transcutaneous oxygen
and CO
2
tensions.
The best answer is B. The third paragraph outlines what the present technology
now does, and what future technology may one day be able to do.

4. It can be inferred from the passage that after "the golden hour has passed,
(A) hypothermia will occur
(B) there is no longer a chance of hypothermia
(C) a patients chances of recovery are diminished
(D) there is little or no chance that hemorrhagic shock will set in.
(E) there is little or no chance of cardiac arrest.

The best answer is C. According to the passage, in order to successfully
resuscitate a person suffering from severe hemorrhage, one must work within
"the golden hour time limit. In other words, after that time limit, the chances of
recovery are lessened.













Historians have traditionally focused on New England as the true
birthplace of America, yet New England was fairly unrepresentative
of the real America - a homogenous society dominated by English
Puritans and their inflexible doctrines. The middle colonies, on the
other hand, were made up of people of diverse origins, races, and
creeds, so their interrelationships are unquestionably more symbolic
of American culture. Like most people's idea of America, the middle
colonies developed a commercial culture based on a balanced
economy, and showed no real homogenous cultural traits that ran
through the region. Indeed, most of the groups that coexisted in
this region did not intermingle with each other, but kept their own
distinctive cultural and social habits. Because of this, the argument
can be made that the middle colonies were not the heterogeneous,
"melting pot" culture that some historians claim existed. After all,
heterogeneous seems to suggest a fusion of different types of
people, when in fact these colonies offered more of a clannish type
of policy when it came to dealing with their new neighbors.

Jack Greene hypothesizes that the idea of mastery was a crucial
factor in shaping America's colonial culture. Greene suggests that
the objective of the English who migrated to the Americas was to
achieve mastery over the rugged land of America as well as of other
groups, a mastery that was unavailable to them in their homeland.
The problem with this hypothesis is that it covers only the English
migration to the New World, and only a relatively small portion of
that group. In truth, many English chose to relocate to America for
reasons having nothing to do with mastery over others, such as
religious freedom and financial gain. Greene also advocates relating
the culture of colonial America back to that of Great Britain, a
technique that can be useful in delineating the differences between
the two areas. However, taking this approach too far can be
deleterious, as the English in America were quick to develop cultural
traits that had little connection to their homeland. A comparison of
some of these cultural aspects would only serve to confuse. Finally,
the many people who came to America from countries besides
England would not fit anywhere into Greene's paradigm.



1. The author mentions Jack Green's hypothesis most likely in
order to

(A) state a conclusion about facts presented in an earlier paragraph
(B) qualify the extent to which a previously mentioned hypothesis
is unsatisfactory
(C) demonstrate that, of three explanatory factors offered, two
may be incorrect
(D) introduce a hypothesis that is considered unsatisfactory by the
author
(E) discredit a previously mentioned hypothesis.

The best answer is D. In paragraph two of the passage, the author
presents Greene's hypothesis along with reasons why it is not
sufficient to understand the history of the America's colonial culture.

2. According to the author, the middle colonies were more
representative of American colonial culture than New England
because

(A) the middle colonies were inhabited by Puritans while New
England was inhabited by diverse creeds and cultures
(B) the middle colonies had symbolic relationships while New
England society was dominant
(C) the middle colonies were peopled by a variety of people from
different races and beliefs while New England was homogeneous
(D) New England was peopled by a variety of people from different
races and beliefs while the middle colonies were homogeneous
(E) New England was inhabited by Puritans while the middle
colonies were inhabited by diverse creeds and cultures


The best answer is C. According to the passage, New England was
a homogenous society while the middle colonies had a
heterogeneous mix of people.

3. Which of the following is NOT cited as an objective of some of
the people that immigrated to America in colonial days?

(A) Religious freedom
(B) Mastery over the land
(C) financial gain
(D) Mastery over other groups
(E) breaking out of a social class

The best answer is E. In paragraph two of the passage, the author
cites A, B, C and D as possible reasons that individuals immigrated
to America.


4. The author's attitude to the claim that the middle colonies were
a heterogeneous, "melting pot" culture is

(A) limited approval
(B) tentative acceptance
(C) mild skepticism
(D) studious criticism
(E) wholehearted endorsement

The best answer is D. The author rejects the claim that the middle
colonies were a heterogeneous "melting pot" culture.

































A computer virus is an illegal and potentially damaging computer
program designed to infect other software by attaching itself to any
software it contacts. In many cases, virus programs are designed to
damage computer systems maliciously by destroying or corrupting
data. If the infected software is transferred to or accessed by
another computer system, the virus spreads to the other system.
Viruses have become a serious problem in recent years, and
currently, thousands of known virus programs exist.

Viruses can be categorized as boot sector viruses, file viruses, and
Trojan horse viruses. A boot sector virus infects the boot program
used to start the system. When the infected boot program executes,
the virus is loaded into the computer's memory. Once a virus is in
the memory, it can spread to any floppy disk inserted into the
computer. The second type of virus, a file virus, inserts viral code
into program files. The virus then spreads to any program that
accesses the infected file. A Trojan horse virus (named after a
Greek myth) hides within or is designed to look like a legitimate
program.

Some viruses interrupt processing by freezing a computer system
temporarily and then displaying sounds or messages. Other viruses
contain time bombs or logic bombs. A time bomb is a program that
performs an activity on a particular date. A logic bomb is a program
that performs an activity when a certain action occurs, such as an
employee being terminated. A worm, which is similar to a virus,
copies itself repeatedly until no memory or disc space remains. To
detect computer viruses, anti-virus programs have been developed.
Besides detecting viruses, anti-virus programs also have utilities to
remove or repair infected programs and files. Some damaged files
cannot be repaired and must be replaced with uninfected backup
files.

Computers can best be protected by installing a virus protection
software. In order to be effective, it is imperative to develop a
regular plan for copying and storing important data and program
files. Prior to running the virus protection software, floppy disks
should be scanned using a virus scan. Since one of the most
vulnerable points of entry for viruses is the internet, all programs
and email messages downloaded from the Internet should be
checked to make sure they are virus free.


1. The passage provides support for which of the following
statements?

A. Computer owners who use virus protection applications properly
are more likely not to suffer damages from computer viruses.
B. Computer owners who use virus protection applications properly
will not suffer damages from computer viruses.
C. Computer owners who use virus protection applications properly
are more likely to suffer damages from computer viruses.
D. Some damaged files can be repaired so must not be replaced
with uninfected backup files.
E. Some damaged files cannot be repaired and must be replaced
with infected backup files.
The best answer is A. The fourth paragraph states that computers
can best be protected by installing virus protection software.

2. According to the passage, a worm is

A. a virus that copies itself repeatedly until no memory or disc
space remains.
B. a virus that cannot be detected by an anti-virus program.
C. something similar to a virus that works by copying itself
repeatedly until no memory or disc space remains.
D. something similar to a virus that uses a logic bomb to performs
an activity when a certain action occurs.
E. an anti-virus program that works by copying itself repeatedly
until no memory or disc space remains.

The best answer is C. Choices A and B are incorrect because a
worm is not a virus. Choice D is incorrect because logic bomb are
not used by worms. Choice E is incorrect because a worm is not an
anti-virus program.



3. According to the passage, computer viruses can spread by which
of the following means?

I. Through boot programs
II. Through floppy disks
III. Through programs downloaded from the Internet

A. I only
B. II only
C. I and II only
D. II and III only
E. I and II and III

The best answer is D. Viruses can affect boot programs but they
cannot spread through boot programs.


4. According to the passage, a virus that inserts viral code into
program files is a
A. Trojan horse virus
B. time bomb
C. worm
D. file virus
E. boot sector virus.

The best answer is D. According to the second paragraph of the
passage, a file virus inserts viral code into program files.






























Until now, efforts at preventing the proliferation of bioweapons have relied on
limiting access to the pathogens themselves. However, if terrorists can produce a
bioweapon from genome information alone, the same logic suggests that access to
genetic information should also be restricted. Scientists have begun to discuss the
restriction of the export of critical pieces of DNA. This measure could be extended to
the three big genome databases.

Scientists have long been aware of the potential dangers. The creation of polio, for
instance, was forecast months ago in an essay in Nature Immunology pointing out that
it would be simple to build an artificial polio virus. Nevertheless, most scientists still
oppose any attempts to restrict access to information. Earlier this year, the US
Department of Defense dropped proposals for checking any research it funds for
"sensitive" information before it is published, after scientists protested that this would
impede research needed to defend against bioweapons.

This was the case with anthrax. Unlike some other potential bioweapons, anthrax still
exists in nature, infecting animals and sometimes humans, and samples are held in
many labs worldwide. There is no reason for a terrorist to try to recreate it. The same
is not true of Ebola, smallpox or the 1918 flu virus. Even so, when the American
Society for Microbiology considered whether it should publish the smallpox genome,
it reasoned that the benefits in terms of understanding the virus and designing drugs
outweighed the risks.

Not all scientists share such views. Raymond Zilinskas of the Monterey Institute of
International Studies in California thinks some limits should be placed on the
publication of information on organisms such as smallpox. Zilinskas contends that
most scientists feel that basic research should not be restricted in this fashion. But
where does one cross the line? He and others have proposed that professional
societies and editorial boards at scientific journals should exert more control.

Even if they do not, the decision could be taken out of their hands. In addition to
international measures, individual governments are also cracking down. The USA
Patriot Act passed this year allows the federal government to stop some foreign
nationals working in the US from getting access to certain pathogens and toxins. The
US could extend this to cover access to genetic sequences as well.



1. The passage is primarily concerned with

(A) detailing the evidence that supports the claim that genetic information should be
restricted

(B) outlining the factors that have contributed to the restriction of genetic information

(C) evaluating whether sensitive genetic information should be restricted in light of
the danger of bioterrorism.

(D) summarizing the differences between genetic research in free countries and such
research in countries that promote terrorism.

(E) formulating a hypothesis about the proliferation of bioweapons.


The best answer is C. The passage does not reach any conclusions, it simply
evaluates the question of limiting access to sensitive genetic information.

2. Terrorists would be unlikely to try to steal genetic information about anthrax
because
(A) it is difficult to control and could therefore be a threat to the terrorists
themselves.
(B) they do not have the necessary know-how to utilize genetic information to build
anthrax powder
(C) actual anthrax is not only available in nature, but also in laboratories.
(D) the American Society for Microbiology has limited the dissemination of genetic
information on anthrax.
(E) the American Society for Microbiology has published genetic information on
anthrax.

The best answer is C. Since anthrax itself is attainable, it is unlikely that bioterrorists
would try to synthesize it using genetic information.

3. All of the following statements are supported by the passage EXCEPT:
(A) the smallpox genome has been published by the American Society for
Microbiology
(B) federal government has stopped some foreign nationals from accessing to
certain toxins
(C) Raymond Zilinskas of the Monterey Institute believes the anthrax genome should
not be published
(D) Most scientists oppose any attempts to restrict access to information
(E) it would not be difficult for a bioterrorist to build an artificial polio virus

The best answer is C. According to the passage, Raymond Zilinskas believes some
limits should be placed on the publication of information, but one cannot determine
from the passage if that would include anthrax information.

4. It can be inferred from the passage that which of the following is true?
(A) The USA Patriot Act will eventually stop all foreign workers from access to
genomes.
(B) It is not known whether or not bioterrorists have to skill to produce an actual
bioweapon from genome information alone.
(C) The Ebola virus affects animals.
(D) According to Nature Immunology magazine, some terrorists have built an
artificial polio virus
(E) The three big genome databases have restricted of the export of critical pieces of
DNA.


The best answer is B. In the first paragraph of the passage it is stated if terrorists
can produce a bioweapon from genome information alone. One can infer from this
that it is not known whether the terrorists can produce one or not.

You might also like